Item: 1 of 15 ~ 1 • M k -<:J 1>- Jil ~· !:';-~ QIO: 3890 ..L a r Previous Next Lab~lu es Notes Calculat o r

• 1 • A 29-year-old female is pregnant with a boy. She was diagnosed with a bleeding disorder when she was a teenager and has recurrent gum bleeding IA•A] . 2 when brushing her teeth. She has anemia secondary to severe menorrhagia. The patient's mother and grandmother h ave had similar problems, but the patent's father has no history of any bleeding disorder. The father of the baby boy has no history of any bleeding disorder either. Laboratory tests • 3 show a platelet count of 250,000/ mm•, increa sed bleeding time and partial thromboplastin time, but normal prothrombin time. · 4 • 5 What Is the probability that the unborn boy will inherit the disease?

• 6 : A. 0% . 7 · 8 B. 25%

. 9 c. 50% • 10 o. 67% • 11 E. 100% • 12 • 13 • 14 • 15

a s 8 Lock Suspend End Block Item: 1 of 15 ~ . I • M k <:] t> al ~· ~ QIO: 3890 .l. ar Previous Next lab 'Vfl1ues Notes Calculator

1 . 2 The correct a nswer is c. 6 1% chose this. • 3 The pregnant woman's history is consistent with von Willebrand disea se (vWD), an autosomal dominant coagulation disorder characterized by mucocutaneous bleeding and often menorrhagia in women. Von Willebrand factor (vWF) is involved in platelet aggregation and is a carrier protein for . 4 factor VIII. Thus, a lack of vW F increa ses the partial thromboplastin time and the bleeding time, wherea s platelet count and prothrombin time remain • 5 normal. We know that the patient is heterozygous for vWD because there was only a history of the bleeding disorder on her mother's side of the family. Because • 6 she has one defective allele and one normal allele, there is a 50% chance that she may pass the defective allele her offspring. Since the father does not . 7 manifest the disea se, he will not pass on a defective allele. Therefore, 50% of the offspring will have 1 defective allele and since it is an autosomal dominant trait, the bleeding disorder will be passed to 50% of her children . • 8 Von Willebrand disease Von Willebrand factor Menorrhagia Prothrombin time Dominance (genetics) Allele Factor VIII Partial thromboplastin time Heterozygous • 9 Platelet Thrombin Autosome Protein Coagulation Coagulopathy Bleeding time Platelet aggregation Mucocutaneous zone • 10 A is not correct. 11% chose this. · 11 There is a 50% chance the unborn child will have the disea se, not 0% . • 12 B is not correct. 13 % chose this • There is a 50% chance the unborn child will have the disea se, not 25% . • 13 D is not correct. 1% chose this • • 14 There is a 50% chance the unborn child will have the disea se, not 67% . • 15 E is not correct. 14 % chose this. There is a 50% chance the unborn child will have the disea se, not 100% .

Botto m Line: Von Willebrand disea se is an autosomal dominant bleeding disorder. The chance of a person contracting an autosomal-dominant disorder that only one parent manifests is 50%, assuming the parent with the defective trait is heterozygous for that tra it. Von Willebrand disease Dominance (genetics) Heterozygous Autosome Coagulopathy Genetic disorder

6 s 0 lock Suspend End Block Item: 1 of 15 ~ . I • M k <:] t> al ~· ~ QIO: 3890 .l. ar Previous Next lab 'Vfl1ues Notes Calculator

1 Bottom Line: . 2 Von Willebrand disease is an autosomal dominant bleeding disorder. The chance of a person contracting an autosomal-dominant disorder that only one • 3 parent manifests is 50%, assuming the parent with the defective trait is heterozygous for that trait. Von Willebrand disease Dominance (genetics) Heterozygous Autosome Coagulopathy Genetic disorder . 4 • 5 • 6 lijl;fiiJI•l toryear:[2017 • ] . 7 FI RST AI D FA CTS • 8 • 9 FA17 p 55.1 Modes of inheritance • 10 Autosomal dominant Often due to defects in structural genes. Many Often pleiotropic (multiple apparently unrelated · 11 generations, both males and females are effects) and variably expressive (different • 12 affected. between individuals). Family history crucial • 13 to diagnosis. With one affected (heterozygous) • 14 parent, on average, Y2 of children affected . • 15

Autosomal recessive Often due to enzyme deficiencies. Usual ly seen Commonly more severe than dominant disorders; in only I generation. patients often present in childhood. t risk in consanguineous famili es. '"'ith 2 ca rrier (heterozygous) parents, on average: !4 of ch il dren wi ll be affected (homozygous), 1/., nf roh ilrlrfHl u ;ill h P 1"'•3t"I";Dt"C "l nrl It;. nr ,..h ;)rlrP I"\ 6 s 0 lock Suspend End Block Item: 1 of 15 ~ 1 • M k -<:J 1>- Jil ~· !:';-~ QIO: 3890 ..L a r Previous Next Lab~lu es Notes Calculat o r

1 p p } p) ••• shows "ragged red fibers" (due to accumulation . 2 of diseased mitochondria). • 3 0 = unaffected male; • = affected male; 0 = unaffected female; e = affected female. · 4 • 5 FA17p 406.1 • 6 Mixed platelet and coagulation disorders . 7 DISORDER PC BT PT PIT MECHANISM AND COMMENTS · 8 von Willebrand t -It Intrinsic pathway coaguJation defect: l v\VF . 9 disease - t PTf (v\VF acts to carry/protect factor • 10 VIII). in platelet plug formation: l v\VF • 11 Defect - defect in platelet-to-v\VF' adhesion . • 12 utosomal dominant. ~ Tild but most common • 13 inherited bleeding disorder. No platelet • 14 aggregation with ristocetin cofactor assay . • 15 Treatment: desmopressin, which releases ,,Wf stored in endothelium. Disseminated t t t Widespread activation of clotting - deli eiency intravascular in clotting fa ctors - bleeding stale. coagulation Causes: (gram 8 ), Trauma, Obstetric com pi ications, acute Pancreatitis, \lalignancy, J\'ephrotic syndrome, Transfusion (STOP ~ l aking ~ ew T hrombi). a s 8 Lock Suspend End Block Item: 1 of 15 ~ 1 • M k -<:J 1>- Jil ~· !:';-~ QIO: 3890 ..L a r Previous Next Lab~lu es Notes Calculat o r 1 .... defect in platelet-to-vWF' adhesion. • • 2 Au tosomal dominant. ~ !ild but most common • 3 inherited bleeding disorder. 'o platelet aggrega ti on with ri stocetin cofactor assay. · 4 Treatment: desmopressin, which releases • 5 ' WF' stored in endothelium . • 6 Disseminated t t t Widespread acti\'ation of clotting .... defi ciency . 7 intravascular in clott ing factors .... bleeding slate. · 8 coagulation Causes: Sepsis (gram 8), Trauma, Obstetric . 9 complications, acute Pancreatitis, phrotic syndrome, • 10 \lalignancy, :\'c Transfusion (STOP ~laking :-.. ew T hrombi). • 11 Labs: schistocrtes, t fibrin degradation • 12 products (o-dimers), ~ fi brinogen, ~ factors • 13 and VII I. • 14 • 15 FA17 p 56.1 Autosomal dominant Achondroplasia, autosomal dominant polycystic kidney disease, fam ilial adenomatous polyposis, diseases familial hypercholesterolemia, heredi tary hemorrhagic telangiectasia, heredita ry spherOC) tosis, Huntington disease, Li-F'raumeni S) ndrome, Jvlarfan syndrome, multiple endocrine neoplasias, neurofibromatosis type l (von Recklinghausen disease), neurofibromatosis type 2, tuberous sclerosis, \'On Hippel-Lindau disease.

a s 8 Lock Suspend End Block Item: 2 of 15 ~ 1 • M k -<:J 1>- Jil ~· !:';-~ QIO: 3190 ..L a r Previous Next Lab~lu es Notes Calculat o r

1 • A 16-year-old girl visits her physician for a routine check-up. She has mild pharyngitis on physical examination, as well as tender axillary . 2 lymphadenopathy. She reports increased tiredness while at school for the past 3 weeks. A shows the following:

• 3 Hemoglobin: 13.0 g/dl Hematocrit: 41% · 4 Mean corpuscular volume: 88 fl • 5 Platelet count: 300,000/ mm• WBC count: 9000/ mm• • 6 A rapid streptococcus test is negative. However, reactive heterophile antibodies that agglutinate horse red blood cells are found in the patient's serum. The . 7 physician also not1ces that large lymphocytes with prominent nucleoli are present In the patient's b shownlood, as in the image. · 8 . 9 • 10

• 11 • 12 • 13 • 14 • 15

a s 8 Lock Suspend End Block • 5 • 6 . 7 · 8 . 9

• 10

• 11 • 12 The virus that causes the infection described above is associated with what other disease? • 13 : • 14 A. Acute glomerulonephritis • 15 B. Cervical carcinoma

C. MALT lymphoma

D. Nasopharyngeal carcinoma

E. Rheumatic heart disease

a s 8 Lock Suspend End Block Item: 2 of 15 ~ . I • M k <:] t> al ~· ~ QIO: 3190 .l. ar Previous Next lab'Vfllues Notes Calculator

1 The correct a nswer is D. 64% chose this. 2 This patient appea rs to be suffering from infectious mononucleosis, most commonly caused by Epstein-Ba rr virus ( EBV). It is characterized classically by fever; malaise, pharyngitis, hepatosplenomegaly, and lymphadenopathy. The peripheral smea r shown in the vignette image indicates atypical lymphocytes • 3 (abnormal circulating cytotoxic T lymphocytes). It is diagnosed by the Monospot test, wherein heterophil antibodies are detected by agglutination of . 4 sheep or horse RBCs. It is transmitted by personal contact, typically through saliva , hence its name "the kissing disea se." EBV infection can be complicated by splenic rupture, and patients should be counseled to avoid contact sports until splenomegaly resolves. EBV infection is associated with • 5 nasopharyngea l carcinoma, which manifests with nasal obstruction with epistaxis, neck mass from metastasis, and hea dache from cranial nerve involvement, typically in an Asian male. EBV infection is also associated with CNS lymphoma and Hodgkin lymphoma . • 6 Infectious mononucleosis Nosebleed Epstein-Barr virus Splenic injury Nasopharynx cancer Splenomegaly Pharyngitis lymphadenopathy Hepatosplenomegaly . 7 lymphocyte Malaise Metastasis Antibody T cell Virus Cranial nerves Headache lymphoma Saliva Central nervous system Hodgkin's lymphoma Spleen • 8 Cytotoxicity Cytotoxic T cell Primary central nervous system lymphoma Carcinoma Pharynx Nasopharynx Infection Fever • 9 A is not correct. 7% chose this • Infectious mononucelosis is not associated with acute glomerulonephritis. Poststreptococcal glomerulonephritis ( PSGN ) is a common acute • 10 glomerulonephritis that occurs after infection with group A streptococcus, most typically a skin infection. Unlike rheumatic hea rt disea se, which also · 11 occurTs after streptococcal infection, it cannot be avoided by antibiotic use. Rheumatic fever Streptococcus Streptococcus pyogenes Antibiotics Glomerulonephritis Acute proliferative glomerulonephritis Group A streptococcal infection • 12 Skin infection Cardiovascular disease

• 13 B is not correct. 7% chose this. • 14 Infection with the human papillomavirus ( HPV) is associated with cervical carcinoma. Epstein-Ba rr virus ( EBV) infection is not associated with an increa sed risk of cervical carcinoma . • 15 Epstein-Barr virus Human papillomavirus Cervical cancer Cervix Carcinoma Virus Infection

c is not correct. 17% chose this. Although some studies have linked Epstein-Ba rr virus ( EBV) infection with an increa sed risk of gastric cancer; EBV is not associated with mucosa-associate lymphoid tissue (MALT) lymphoma. Infection with Helicobacter pylori is associated with peptic ulcer disea se and MALT lymphoma. Peptic ulcer Epstein-Barr virus Helicobacter pylori MALT lymphoma Stomach cancer lymphoma lymphatic system Cancer Virus Ulcer Infection Ulcer {dermatology)

E is not correct. 5 % chose this. This is a case of mononucleosis, typically caused by Epstein-Ba rr virus ( EBV). Rheumatic hea rt disea se is associated with untrea ted group A streptococcal infPrtinn mnc;t r nmmnnlv nh ;::~ r·vnniti~ R hP 11m ;::~ ti r h P;::t tT rlic;:P;::t c;:P r;::~ n h P nr·PvPntPrl hv tno;::~ tmP nt with ;::~ ntihinti rc;: 6 s 0 lock Suspend End Block Item: 2 of 15 ~ . I • M k <:] t> al ~· ~ QIO: 3190 .l. ar Previous Next lab'Vfllues Notes Calculator

1 • • • • lymphoid tissue (MALT) lymphoma. Infection with Helicobacter pylori is associated with peptic ulcer disea se and MALT lymphoma. 2 Peptic ulcer Epstein-Barr virus Helicobacter pylori MALT lymphoma Stomach cancer lymphoma lymphatic system Cancer Virus Ulcer Infection • 3 Ulcer {dermatology) . 4 E is not correct. 5 % chose this. This is a case of mononucleosis, typically caused by Epstein-Ba rr virus ( EBV). Rheumatic hea rt disea se is associated with untrea ted group A streptococcal • 5 infection, most commonly pharyngitis. Rheumatic hea rt disea se can be prevented by trea tment with antibiotics. • 6 Epstein-Barr virus Infectious mononucleosis Rheumatic fever Pharyngitis Group A streptococcal infection Streptococcus Virus Cardiovascular disease Antibiotics . 7 • 8 Bottom Line: • 9 Epstein-Ba rr virus ( EBV) is the most common cause of infectious mononucleosis and is known as the "kissing disea se" because of transmission through saliva. Positive heterophil antibodies is specific for EBV infection. Infection with EBV is associated with nasopharygea l carcinoma, CNS lymphoma, • 10 Hodgkin lymphoma, and splenic rupture. Infectious mononucleosis Epstein-Barr virus Splenic injury lymphoma Antibody Virus Hodgkin's lymphoma Central nervous system Saliva Spleen · 11 Primary central nervous system lymphoma Carcinoma • 12 • 13 • 14 lijj ;fi IJ l•l for year:l 2017 .. FIRST AID FACT S • 15

FA17 p 160.2 Herpesviruses Enveloped, OS, and linear viruses

VIRUS ROUTE OF TRANSMISSION CLINICAL SIGNIFICANCE NOTES Herpes Respiratory Gingivos tomatitis, keratoconjunctivitis fJ, lost common cause of sporadic simplex secretions, saliva herpes labial is rn. herpetic wh itlow on finger, encephalitis, can present as altered virus-1 temporal lobe encephalitis, esophagitis, mental status, seizures, and/or 6 s 0 lock Suspend End Block Item: 2 of 15 ~ 1 • M k -<:J 1>- Jil ~· !:';-~ QIO: 3190 ..L a r Previous Next Lab~lu es Notes Calculat o r

- . - -~ - .... -~ -=-= =- = k - ~ 1 •

2 FA17 p 160.1 • 3 DNA viruses · 4 VIRAl FAMILY ENVELOPE DNA STRUCTURE MEDICAL IMPORTANCE • 5 Herpesviruses Yes DS and linear See ll c rp c~'· iruses entry • 6 Poxvirus Yes DS and linear Smallpox eradicated world'' ide by usc of the li,c­ . 7 (largest o, 'irus) allcnualcd ,·accine · 8 Co" pox ("milkmaid blisters") lorcd papule'' ith . 9 Molluscum contagiosum-Aesh-co central umbilication • 10 Hepadnavirus Yes Partially DS and circular IIBV: • 11 Acute or chronic hepatitis • 12 1 ot a retrovirus but has reverse transcriptasc • 13 Adenovirus 1 0 DS and linc<1r Febri le pharyngitis rl-sore throat • 14 Acute hemorrhagic cystitis • 15 Pneumonia Conj1mctivit is-"pink eye"

Papillomavirus DS and circular II PV- warts (serot)-pes l. 2, 6, ll), CI '·cervical ______, ___ .._ ------L- 1£ 1 0 \ a s 8 Lock Suspend End Block Item: 3 of 15 ~ 1 • M k -<:J 1>- Jil ~· !:';-~ QIO: 4698 ..L a r Previous Next Lab~lu es Notes Calculat o r

1 • A 2-year-old girl is brought to the emergency department because of fevers that have been occurring daily for 3 weeks. Physical examination reveals lA• A] 2 a 1.2-kg (2.6-lb) weight loss since her last doctor's visit 1 month earlier. She also has pallor, hepatomegaly, and splenomegaly. She is admitted to the hospital and undergoes bone marrow aspiration. Results are shown in the Image. • 3 · 4 • 5 • 6 . 7 · 8 . 9 • 10

• 11 • 12 • 13 • 14 • 15

Which of the follow1nq is the most likely diaqnosis? a s 8 Lock Suspend End Block • 10

• 11 • 12 Which of the following is the most likely diagnosis? • 13 : • 14 A. Ewing sarcoma • 15 B. Hemophagocytic lymphohistiocytosis

c. Multiple myeloma

D. Neuroblastoma

E. Wilms tumor

a s 8 Lock Suspend End Block Item: 3 of 15 ~ . I • M k <:] t> al ~· ~ QIO: 4698 .l. ar Previous Next lab 'Vfl1ues Notes Calculator

1 The correct a nswer is B. 55% chose this. 2 This patient has symptoms consistent with hemophagocytic lymphohistiocytosis (HLH), and bone marrow aspiration (demonstrating macrophages engulfing RBCs) confirms the diagnosis. Fa milial-type HLH is an autosomal recessive defect in several genes, including perforin-related genes; the 3 secondary type is frequently due to infection with Epstein-Ba rr virus. HLH involves the abnormal activation and proliferation of lymphohistiocytes, lea ding . 4 to hemophagocytosis and the upregulation of proinflammatory cytokines. The immune system becomes overstimulated and can begin attacking native cells in the bone marrow (as seen in the vignette image), the liver; and brain. Trea tment consists of a combination of etoposide, corticosteroids, and • 5 methotrexate, although stem cell transplant is often required. Etoposide Epstein-Barr virus Methotrexate Hemophagocytic lymphohistiocytosis Bone marrow examination Bone marrow Corticosteroid liver Immune system • 6 Macrophage Cytokine Hematopoietic stem cell transplantation Bone Stem cell Brain Autosomal recessive Dominance (genetics) Virus . 7 A is not correct. 1 2 % chose this • • 8 Ewing sarcoma frequently presents in the second decade of life with constitutional symptoms (fever; weight loss), bone tenderness, and pathologic • 9 fractures. Bone marrow aspiration would not demonstrate hemophagocytosis. Bone marrow examination Ewing's sarcoma Bone marrow Weight loss Sarcoma Fever Bone • 10 c is not correct. 1 6 % chose this. · 11 Although multiple myeloma frequently manifests with constitutional symptoms, a bone marrow aspirate would show red blood cells in formation, • 12 stacks of RBCs pushed together by excessive protein, along with excessive white blood cells in the form of plasma cells. Multiple myeloma is very rare in children. • 13 Multiple myeloma Rouleaux Protein White blood cell Blood plasma Bone marrow Bone marrow examination Bone

• 14 D is not correct. 9 % chose this • • 15 Neuroblastoma frequently manifests with constitutional symptoms (fever; weight loss, pallor; hepatosplenomegaly). It is the most common malignancy of infancy. A bone marrow aspirate would not demonstrate hemophagocytosis. Neuroblastoma Hepatosplenomegaly Weight loss Bone marrow Hemophagocytosis Bone marrow examination Malignancy Fever Pallor Cancer Bone

E is not correct. 8% chose this. Wilms tumor frequently manifests with an asymptomatic flank mass and/or gross hematuria. It is the most common primary renal tumor in children. Bone marrow aspiration would not demonstrate hemophagocytosis. Hematuria Wilms' tumor Bone marrow examination Neoplasm Bone marrow Asymptomatic Bone

Botto m Line: 6 s 0 lock Suspend End Block Item: 3 of 15 ~ - I • M k <:] t> al ~· ~ QIO: 4698 .l. ar Previous Next lab 'Vfl1ues Notes Calculator

1 E is not correct. 8% chose this. Wilms tumor frequently manifests with an asymptomatic flank mass and/or gross hematuria. It is the most common primary renal tumor in children. Bone 2 marrow aspiration would not demonstrate hemophagocytosis. Hematuria Wilms' tumor Bone marrow examination Neoplasm Bone marrow Asymptomatic Bone 3 . 4 • 5 Bottom Line: • 6 Hemophagocytic lymphohistiocytosis can be identified on a bone marrow smear by looking for macrophages engulfing RBCs. Treatment involves etoposide, methotrexate, and corticosteroids. . 7 Etoposide Methotrexate Hemophagocytic lymphohistiocytosis Corticosteroid Macrophage Bone marrow Bone • 8 • 9 • 10 I ill ;fi 1!1 I•J for year:[ 2017 .. FI RST AID FACTS · 11

• 12 FA17 p412.2 • 13 Langerhans cell Collective group of prol iferative disorders of • 14 histiocytosis dendritic (Langerhans) cells. Presents in a • 15 child as lytic bone lesions rl and skin rash or as recurrent otitis media with a mass involving the mastoid bone. Cells are functionally immature and do no t effectively stimulate primary T cells via antigen presentation. Cells express S-100 (mesodermal origin) and COla. Birbeck granules ("tennis rackets" or rod shaped on EM) are characteri stic [l). 6 s 0 lock Suspend End Block Item: 3 of 15 ~ 1 • M k -<:J 1>- Jil ~· !:';-~ QIO: 4698 ..L a r Previous Next Lab~lu es Notes Calculat o r - - - - - . . . . . • 1 rod shaped on ENf) are characteri stic IJ, 2 3 FA17 p 409.1 · 4 Multiple myeloma lonoclonal plasma cell ("fried egg" Think CR\B: • 5 appearance) cancer that arises in the marrow HyperCalcemia • 6 M Spike ...;• and produces la rge amounts of lgC (55%) or Renal i1wolvement . 7 IgA (25%). Bone marro\\ > 10% monoclonal Anemia · 8 plasma cells. ~ l ost common 1° tumor arising Bone lytic lesions/Back pain rears old. . 9 within bone in people> 40-50 \lultiple \ lyeloma: \ lonoclonal \I protein Associated with: spike • 10 • t susceptibility to infection Distinguish from Waldenstrom • 11 • Primary amyloidosis (AL) macroglobulinemia ..... Nl spike= IgM • 12 Punched-out lytic bone lesions on x-ray fJ ..... hypen·iscosity syndrome (cg, blurred \'ision, • 13 M spike on serum protein electrophoresis Raynaud phenomenon); no ''CRAB" findings . • 14 • lg light chains in urine (} • 15 Rouleaux formation 1J (RBCs slacked like poker chips in blood smear) lumerous plasma cells "ith "clock-face" chromatin and intracytoplasm ic inclusions containing immunoglobulin. Monoclonal gammopathy of undetermined significance (MGUS)-monoclonal expansion of plasma cells (bone marrow< 10% a s 8 Lock Suspend End Block Item: 3 of 15 ~ 1 • M k -<:J 1>- Jil ~· !:';-~ QIO: 4698 ..L a r Previous Next Lab~lu es Notes Calculat o r

1 • 2 3 · 4 • 5 • 6 . 7 · 8 . 9 • 10 • 11 FA17 p 569.1 • 12 Nephroblastoma t.. lost common re nal malignancy of early childhood (ages 2- 4). Contains embryonic glomerular • 13 (Wilms tumor) structures. Presents with large, palpable, un ilateral nank mass · and /or hematuria. • 14 "Loss of function" mutations of tumor suppressor genes \V'J'l or WT2 on chromosome 11. • 15 May be a part of several sy ndromes: \VACR complex: Wilms tumor, Aniridia (absence of iris), Genitourinary malformations, mental Retardation/intellectw1 l disabi lit y (WTI deletion) • Denys-Drash: Wilms tumor, early-onset nephrotic syndrome, male pseudohermaphroditism (WTl mutation) Beckwith-Wiedemann: Wilms tumor, mac rogl o~s i a, organomcgaly, hemihyperplasia (WT2 mutati on)

a s 8 Lock Suspend End Block 4 15 • M k £!1}>' !!":-~ Item: of ~ 1 a r -<:J I> fJ • QIO: 4730 ..L Previous Next Lab lues Notes Calculat o r

1 • The blood smear of a 14-year-old anemic girl is shown in the image. 2 3 . 4 • 5 • 6 . 7 · 8 . 9 • 10

• 11 • 12 • 13 • 14 • 15 0 • - • Which of the following is the most likely pathogenesis of this girl's anemia?

a 8 Lock Suspend End Block Item: 4 of 15 ~ 1 • M k -<:J 1>- Jil ~· !:';-~ QIO: 4730 ..L a r Previous Next Lab~lu es Notes Calculat o r

1

2 3 0 . 4 0 • 5 • 6 • . 7 · 8 . 9 • 10 • 11 • • 12 • Which of the following is the most likely pathogenesis of this girl's anemia? • 13

• 14 A. Glucose-6-phosphate dehydrogenase deficiency • 15 B. Mutation in the phosphatidylinositol glycan gene

c. Mutation in the spectrin gene

D. Physical disruption of otherwise normal RBCs

E. Point mutations of one of the (3-globin genes

a s 8 Lock Suspend End Block Item: 4 of 15 ~ . I • M k <:] t> al ~· ~ QIO: 4730 .l. ar Previous Next lab 'Vfl1ues Notes Calculator

1 The correct a nswer is c. 76% chose this. 2 Spherocytes are evident in this patient's blood smea r; therefore, she most likely suffers from hereditary . In this condition the RBC membrane is not as flexible due to mutations in either of the genes that code for its specialized membrane cytoskeleton. Common gene mutations that 3 lea d to this disorder include spectrin, ankyrin, band 3, or protein 4.2. Because these RBCs are not as flexible, when they go through the sinusoids of the spleen they get trapped there and destroyed. Therefore, splenectomy is currently the best option available to correct this hemolytic anemia. 4 Splenectomy Spectrin Protein 4.2 Cytoskeleton Gene Hemolytic anemia Anemia Blood film Spleen Ankyrin Protein Spherocytosis • 5 Hemolysis Mutation Red blood cell Cell membrane • 6 A is not correct. 7% chose this. . 7 Although infection or exposure to certain drugs in the context of glucose-6-phosphate dehydrogenase deficiency can lea d to a hemolytic anemia, one would not expect to see spherocytes in this condition. Bite cells may be evident in blood smea rs of these patients, as well as RBCs with Heinz bodies • 8 (precipitates of denatured globin). • 9 Glucose-6-phosphate dehydrogenase Hemolytic anemia Glucose-6-phosphate dehydrogenase deficiency Degmacyte Anemia Spherocytosis Hemolysis Glucose 6-phosphate Infection Complete blood count Blood film Globin • 10 B is not correct. 6 % chose this. · 11 Mutations in PIGA lea d to the inability to synthesize glycosyl phosphatidyl inositol (GPI), which is an anchor for several membrane-associated proteins. • 12 These mutations lea d to paroxysmal nocturnal hemoglobinuria because three GPI-Iinked proteins that are regulators of complement activity are deficient, and the RBCs (as well as granulocytes and platelets) are lysed by the spontaneous activa tion of the complement system . One would not expect to see • 13 spherocytes in this condition. • 14 Paroxysmal nocturnal hemoglobinuria Complement system Phosphatidylinositol PIGA Granulocyte Hemoglobinuria Platelet lysis Inositol Membrane protein Spherocytosis Protein • 15 D is not correct. 5 % chose this. Physical damage to RBCs can occur in the context of prosthetic cardiac valves or narrowing of the microvasculature (as in disseminated intrava scular coagulation, malignant hypertension, or systemic lupus erythematosus). One would expect to see (fragments of RBCs) in the peripheral blood smea r; as well as "helmet cells" or "triangle cells," in these cases. Disseminated intravascular coagulation Systemic lupus erythematosus Hypertensive emergency Hypertension Blood film Coagulation Heart valve Malignancy lupus erythematosus Cancer Blood vessel

E is not correct. 6 % chose this. Point mutations of j3 -globin genes are responsible for j3 -thalassemia minor. Pa tients with this disea se have a mild microcytic anemia and do not have c:::;nhpr·nrvtP c:::; in thPir· hlnnrl c:::;m p;::~ r·c:::; 6 s 0 lock Suspend End Block Item: 4 of 15 ~ . I • M k <:] t> al ~· ~ QIO: 4730 .l. ar Previous Next lab 'Vfl1ues Notes Calculator

I I I 1 smea r; as well as "helmet cells" or "triangle cells," in these cases. 2 Disseminated intravascular coagulation Systemic lupus erythematosus Hypertensive emergency Hypertension Blood film Schistocyte Coagulation Heart valve 3 Malignancy lupus erythematosus Cancer Blood vessel 4 E is not correct. 6 % chose this. Point mutations of j3 -globin genes are responsible for j3 -thalassemia minor. Patients with this disease have a mild microcytic anemia and do not have • 5 spherocytes in their blood smears. • 6 Microcytic anemia Anemia Mutation Spherocytosis Point mutation Complete blood count Blood film . 7 • 8 Bottom Line: • 9 In hereditary spherocytosis, the RBC membrane is less flexible because of mutations in membrane cytoskeleton. These RBCs are trapped and destroyed by the spleen, causing hemolytic anemia. Therefore, splenectomy is currently the best option ava ilable . • 10 Hereditary spherocytosis Splenectomy Cytoskeleton Anemia Spherocytosis Spleen Hemolytic anemia Hemolysis Cell membrane Red blood cell Mutation · 11 • 12 • 13 I ill ;fi 1!1 I•J for year:l 2017 .. FI RST AI D FACTS • 14

• 15 FA17 p400.1 Intrinsic hemolytic anemia DESCRIPTION FINDINGS Hereditary Extravascular hemolysis due to defect in Splenomegaly, aplastic crisis (parvovirus Bl9 spherocytosis proteins interac ting with RBC membrane infectio n). skeleton and plasma membrane (eg, ankyrin, Labs: osmotic fragility test®. lormal to band 3, protein 4.2, spectrin). lastly l MCV wi th abundance of cells. autosomal dominant inheritance. Treatment: splenectomy. 6 s 0 lock Suspend End Block Item: 5 of 15 ~ 1 • M k -<:J 1>- Jil ~· !:';-~ QIO: 3461 ..L a r Previous Next Lab~lu es Not es Calculat o r

1 • A 35-year-old woman presents to her physician with complaints of unusually heavy menstrual flow and recurrent nosebleeds that take hours to stop 2 on their own. She says that her mother also had m enorrhagia. Review of systems is otherwise negative. Laboratory tests show:

3 WBC count: 9000/mm• Hematocrit: 36% 4 Platelet count: 300,000/mm• • 5 Bleeding time: 13 m1nutes Prothrombin time: 11 seconds • 6 Partial thromboplastin time: 67 seconds

. 7 Vital signs and baSic laboratory analysis induding chemistries are within normal limits. · 8 . 9 What Is this patient's most likely diagnosis?

• 10 : Bernard-Soulier disease • 11 A.

• 12 B. Disseminated intravascular coagulation • 13 c. Hemophilia A • 14 D. Idiopathic thrombocytopenic purpura • 15 E. Thrombotic thrombocytopenic purpura

F. Von Willebrand's disease

a s 8 Lock Suspend End Block Item: 5 of 15 ~ . I • M k <:] t> al ~· ~ QIO: 3461 .l. ar Previous Next lab 'Vfl1ues Notes Calculator

1 The correct a nswer is F. 66% chose this. 2 This patient has von Willebrand's disea se, the most common bleeding disorder; with an estimated preva lence of 1% . It often presents with increa sed bleeding from mucous membranes in the presence of a normal platelet count and is caused by a low von Willebrand factor (vWF) level, which functions to 3 carry and protect factor VIII. As a result, there is a defect in both platelet adhesion and factor VIII survival. Therefore, von Willebrand's disea se results in increa sed bleeding time and an increa sed partial thromboplastin time. Because vW F is responsible for protecting factor VIII from destruction, a low vW F 4 level often presents with some elements of hemophilia, but it does not progress to the severe joint bleeds commonly seen in patients with hemophilia. It 5 is generally inherited as an autosomal-dominant trait, with variable penetrance. Von Willebrand disease Von Willebrand factor Haemophilia Partial thromboplastin time Penetrance Platelet Factor VIII Coagulopathy Bleeding time • 6 Genetic disorder Dominance (genetics) . 7 A is not correct. 7 % chose this. • 8 Bernard-Soulier disea se is a platelet disorder characterized by low glycoprotein Ib, which, as in von Willebrand's disea se, results in poor platelet-to­ collagen adhesion. However; Bernard-Soulier disea se is characterized by low platelet count . • 9 Von Willebrand disease Glycoprotein Platelet Glycoprotein Ib Thrombocytopenia Bernard-Soulier disease • 10 B is not correct. 3 % chose this. · 11 Disseminated intrava scular coagulation is a systemic condition that is usually the result of sepsis or trauma. It is characterized by the widesprea d activation of the clotting cascade. Patients show decrea sed platelet level and increa sed bleeding, prothrombin, and partial thromboplastin times . • 12 Disseminated intravascular coagulation Sepsis Thrombin Platelet Coagulation Thromboplastin Blood vessel • 13 c is not correct. 1 5 % chose this • • 14 Hemophilia A results from a congenital defect in factor VIII levels or activity. It is an X-linked disea se, so it rarely presents in females. It is associated with traumatic joint and intra cranial bleeds and increa sed partial thromboplastin time but normal bleeding time . • 15 Haemophilia Haemophilia A Factor VIII Partial thromboplastin time Congenital disorder Sex linkage Bleeding time

D is not correct. 5 % chose this. Idiopathic thrombocytopenic purpura is an autoimmune disea se that is caused by antibodies directed against platelet antigens. It normally presents with mucous membrane bleeds, but, unlike von Willebrand disea se, platelet levels are reduced to very low levels (often < 10,000/mm• ). Mucous membrane Von Willebrand disease Immune thrombocytopenic purpura Autoimmune disease Platelet Purpura Antibody Autoimmunity Idiopathy Antigen

E is not correct. 4 % chose this. Thrombotic thrombocytopenic purpura is classically tested on the boards. Remember the pentad of thrombocytopenia, hemolytic anemia, fever; renal impairment, and neurologic symptoms. Thrombotic thrombocytopenic purpura Thrombocytopenia Anemia Purpura Hemolytic anemia Thrombosis Hemolysis Neurology Fever Kidney 6 s 0 lock Suspend End Block Item: 5 of 15 ~ . I • M k <:] t> al ~· ~ QIO: 3461 .l. ar Previous Next lab 'Vfl1ues Notes Calculator

1 Idiopathic thrombocytopenic purpura is an autoimmune disea se that is caused by antibodies directed against platelet antigens. It normally presents with mucous membrane bleeds, but, unlike von Willebrand disea se, platelet levels are reduced to very low levels (often < 10,000/mm• ). 2 Mucous membrane Von Willebrand disease Immune thrombocytopenic purpura Autoimmune disease Platelet Purpura Antibody Autoimmunity Idiopathy Antigen

3 E is not correct. 4 % chose this. 4 Thrombotic thrombocytopenic purpura is classically tested on the boards. Remember the pentad of thrombocytopenia, hemolytic anemia, fever; renal impairment, and neurologic symptoms. 5 Thrombotic thrombocytopenic purpura Thrombocytopenia Anemia Purpura Hemolytic anemia Thrombosis Hemolysis Neurology Fever Kidney • 6

. 7 Bottom Line: • 8 Von Willebrand's disea se is characterized by low vW F levels, resulting in low factor VIII and poor platelet-collagen adhesion. Affected patients may note menorrhagia and mucous membrane bleeding such as epistaxis or prolonged bleeding during dental procedures. Labs are notable for increa sed bleeding • 9 time and partial thromboplastin time. • 10 Von Willebrand disease Nosebleed Menorrhagia Mucous membrane Partial thromboplastin time Factor VIII Bleeding time Von Willebrand factor · 11 • 12 I ill ;fi 1!1 I•J for year:[ 2017 .. • 13 FI RST AID FACTS • 14 • 15 FA17 p406.1 Mixed platelet and coagulation disorders DISORDER PC BT PT PTT MECHANISM AND COMMENTS von Willebrand t - It Intrinsic pathway coagulation defect: l vWF disease ..... t PTT (vWF acts to carry/protect factor V JII). Defect in platelet plug formation: l vWF ..... defect in platelet-to-vWF' adhesion. 6 s 0 lock Suspend End Block Item: 5 of 15 ~ . I • M k <:] t> al ~· ~ QIO: 3461 .l. ar Previous Next lab 'Vfl1ues Notes Calculator

1 intravascular in clotting factors -+ bleeding stale. 2 coagulation Causes: Sepsis (gram 8), T ra uma, O bstetric 3 complications, acute Pancreatitis, Malignancy, Nephrotic syndrome, 4 Transfusion (STOP Making New T hrombi). 5 Labs: schistocytes, t fibr in degradation • 6 products (o-d imers), ~ fi brinogen, ~ factors V . 7 and VIII. • 8 • 9 FA17 p404.2 • 10 Coagulation disorders PT - tests fu nction of common and extrinsic pathway (factors I, II, V, VII, and X). Defect -+ t PT · 11 I lR (international normali zed ratio)-calculated from PT. I = normal,> I = prolonged. Most • 12 common test used to follow patients on warfarin . • 13 PTT-tests function of common and intrins ic pathway (a ll factors except V II and XIII). Defect • 14 __. t PIT. Coagulation disorders can be due to clotting factor deficiencies or acquired inhibitors. Diagnosed • 15 with a mixing study, in which normal plasma is added to patient's plasma. Clotting factor deficiencies should correct (the PT or PIT returns to within the appropriate normal range), whereas factor inhibitors will not correct.

DISORDER PT PTT MECHANISM AND COMMENTS Hemophilia A, B, or C t Intrinsic pathway coagulation defect. • A: deficiency of factor VIII -+ t PTT; X-I inked recessive. • B: deficiency of factor IX -+ t PTT; X-Iinked recessive. 6 s 0 lock Suspend End Block Item: 5 of 15 ~ 1 • M k -<:J 1>- Jil ~· !:';-~ QIO: 3461 ..L a r Previous Next Lab~lu es Not es Calculat o r

1 Vitamin K deficiency t f General coagulation defect. Bleeding lime normal. • 2 l activity of factors II, VII , lX, X, protein C, proteinS. 3 4 FA17 p 405.1 5 Platelet disorders Defects in platelet plug formation - f bleeding time (BT). • 6 Platelet abnormalities - mierohemorrhage: mucous membrane bleeding, epistaxis, petechiae, . 7 purpura, f bleeding time, possibly decrea ed platelet count (PC).

· 8 DISORDER PC BT MECHANISM AND COMMENTS . 9 Bernard-Soulier - ll f Defect in platelet plug formal ion. Large platelets. • 10 syndrome l Cplb - defect in platelel-to-v\VF adhesion. • 11 Glanzmann f Defect in platelet inlegrin a11 h~ (Cpll b/llla) - defect in platelet-to-platelet • 12 thrombasthenia aggregation, and therefore platelet plug formation. • 13 Labs: blood smear sho"s no platelet clumping . • 14 Hemolytic-uremic f Characterized by thrombocytopenia, microangiopathic hemolytic anemia, and syndrome acute rena l failure. • 15 Typical HUS is seen in chi ldren, accompanied by diarrhea and commonly caused by entcrohcmorrhagic E coli (E I-I F.C) (eg, 0157:1-1 7). I-IUS in adu ll s docs not present wit h dit~rrh e~l i El l EC infection not required. Same spectrum as T I'P, with a si milar clinical presentation and same initial treatment of plasmapheresis. Immune f Anti-Cpll b/llla antibod ies - splenic macrophage consumption of thrombocytopenia platelet-antibody complex. 1\ lay be 1° (idiopathic) or zoto autoimmune ,. . 1 • 11 1 . a s 8 Lock Suspend End Block Item: 6 of 15 ~ 1 • M k -<:J 1>- Jil ~· !:';-~ QIO: 3856 ..L ar Previous Next Lab~lues Notes Calculator

1 • An 86-year-old man presents to his physician with 4 months of increasing fatigue, a 6 .8-kg (15-lb) unintentional weight loss, and an uncomfortable 2 sense of fullness in the left upper quadrant. A complete blood cell count shows:

3 WBC count: 14,000/mm• Hemoglobin: 9 g/dl 4 Hematocrit: JOo/o 5 Platelet count: 470,000/mm• • 6 . 7 · 8 . 9 • 10

• 11 • 12 • 13 • 14 • 15

a s 8 Lock Suspend End Block · 8 . 9

• 10 Image courtesy of Paulo Henrique Orlandi Mourao • 11 • 12 Which of the following processes is associated with this patient's condition? • 13 : • 14 A . Aplastic anemia • 15 8. Asplenia

c. Hemoglobin clumping

o. Intravascular hemolysis

E. Myelofibrosis

a s 8 Lock Suspend End Block Item: 6 of 15 ~ 1 • M k -<:J 1>- Jil ~· !:';-~ QIO: 3856 ..L ar Previous Next Lab~lues Notes Calculator • 1 The c o rrect answ er i s E. 700/o chose this. 2 This patient most likely has idiopathic myelofibrosis, which is a myeloproliferative syndrome in which bone marrow Is gradually replaced by collagen fibrosis. These fibrotic changes explain the unsuccessful attempts 3 at obtaining a bone marrow aspiration. Laboratory findings often include anemia, mild thrombocytosis, and laboratory values. In response to gradual bone marrow failure, 4 mild neutrophilia, as seen in this patient's extramedullary hematopoiesis begins to occur in locations such as the spleen, resulting in splenomegaly, 5 which accounts for the patient's upper quadrant discomfort. It is believed that myelofibrosiS results in distortion and fragmentation of the cell membrane, yielding (also spelled dacryocytes), which 6 are teardrop-shaped RBCs, on the peripheral blood smear, indicated by the arrows in this image. A number of genes assoc1ated with primary myelofibrosis, induding JAK2, CALR, TET2, and MPL. 0 7 Myl!lofibrosis E tramedullary hematopoiesis Janus kinase 2 Myeloproliferat•ve neoplasm o8 I Splenomegaly Blood film Bone marrow examination Haematopoiesis Collagen Anemia Thrombocytosis . 9 Bone marrow Spleen Cell membrane Fibrosis Aplastic anemia Calreticulin Idiopathy Bone • 10 Image courtesy of Paulo Henrique Orlandi • 11 Mourao • 12 A i s not co rrect. 120/o chose this• • 13 Aplastic anemia results in pancytopenia, but normochromic and normocytic RBCs. Aplastic anemia Pancytopenia Anemia Normocytic anemia 0 14 • 15 B is not co rrect. 60/o chose this. Target cells can be present in a splenia a nd both a- and j3 -thalassemla . Asplenia

C is not co rrect. 40/o chose this. Hemoglobin clumping, commonly found in glucose-6-phosphate dehydrogenase deficiency, appears as Heinz bodies on peripheral smear. Heinz body Hemoglobin Glucose-6-phosphate dehydrogenase Glucose-6-phosphate dehydrogenase deficiency Glucose 6-phosphate

0 is not correct. 80/o chose this. Intravascular hemolysis, such as that caused by disseminated intravascular coagulation, would result in the appearance of helmet cells and schlstocytes. Disseminated intravascular coagulation Hemolysis Schistocyte Hemolytic anem1a Coagulation Blood vessel a s 8 Lock Suspend End Block Item: 6 of 15 ~ . I • M k <:] t> al ~· ~ QIO: 3856 .l. ar Previous Next lab 'Vfl1ues Notes Calculator

1 c is not correct. 4 % chose this. 2 Hemoglobin clumping, commonly found in glucose-6-phosphate dehydrogenase deficiency, appears as Heinz bodies on peripheral smear. Heinz body Hemoglobin Glucose-6-phosphate dehydrogenase Glucose-6-phosphate dehydrogenase deficiency Glucose 6-phosphate 3 D is not correct. 8% chose this. 4 Intrava scular hemolysis, such as that caused by disseminated intravascular coagulation, would result in the appearance of helmet cells and schistocytes. 5 Disseminated intravascular coagulation Hemolysis Schistocyte Hemolytic anemia Coagulation Blood vessel 6

. 7 Bottom Line: • 8 In response to myelofibrosis, bone marrow hematopoesis may be obliterated, resulting in compensatory extramedullary hematopoiesis, which may take place in the liver; spleen, and lymph nodes. On peripheral blood smear; dacrocytes, or teardrop-shaped RBCs, are a sign of myelofibrosis as a result of • 9 physical damage sustained from squeezing of RBCs out of increasingly fibrotic bone marrow. Myelofibrosis Extramedullary hematopoiesis Haematopoiesis Blood film Spleen Bone marrow liver Fibrosis lymph node Bone lymph • 10 · 11 • 12 lijj ;fi IJ l•l for year:l 2017 .. • 13 FI RST AI D FACTS • 14 • 15 FA17 p411.1 Chronic The myeloproliferati ve disord ers (polycythemia vera, essential thrombocythemia, myelofibrosis, and myeloproliferative C.\11L) are malignant hematopoietic neoplasms with varying impacts on WBCs and myeloid cel l disorders lines. Associated with V617F' jAK2 mutation. Polycythemia vera Primary polycythemia. Disorder of t RBCs. May present as intense itching after hot shower. Rare but classic symptom is erythromelalgia (severe, burning pa in and red-blue coloration) due to episodic blood clots in vessels of the extremities a. t EPO {vs 2° polycythemia, which presents with endogenous or artificially t EPO). 6 s 0 lock Suspend End Block Item: 6 of 15 ~ . I • M k <:] t> al ~· ~ QIO: 3856 .l. ar Previous Next lab 'Vfl1ues Notes Calculator

1 c is not correct. 4 % chose this. 2 Hemoglobin clumping, commonly found in glucose-6-phosphate dehydrogenase deficiency, appears as Heinz bodies on peripheral smear. Heinz body Hemoglobin Glucose-6-phosphate dehydrogenase Glucose-6-phosphate dehydrogenase deficiency Glucose 6-phosphate 3 D is not correct. 8% chose this. 4 Intrava scular hemolysis, such as that caused by disseminated intravascular coagulation, would result in the appearance of helmet cells and schistocytes. 5 Disseminated intravascular coagulation Hemolysis Schistocyte Hemolytic anemia Coagulation Blood vessel 6

. 7 Bottom Line: • 8 In response to myelofibrosis, bone marrow hematopoesis may be obliterated, resulting in compensatory extramedullary hematopoiesis, which may take place in the liver; spleen, and lymph nodes. On peripheral blood smear; dacrocytes, or teardrop-shaped RBCs, are a sign of myelofibrosis as a result of • 9 physical damage sustained from squeezing of RBCs out of increasingly fibrotic bone marrow. Myelofibrosis Extramedullary hematopoiesis Haematopoiesis Blood film Spleen Bone marrow liver Fibrosis lymph node Bone lymph • 10 · 11 • 12 lijj ;fi IJ l•l for year:l 2017 .. • 13 FI RST AI D FACTS • 14 • 15 FA17 p411.1 Chronic The myeloproliferati ve disord ers (polycythemia vera, essential thrombocythemia, myelofibrosis, and myeloproliferative C.\11L) are malignant hematopoietic neoplasms with varying impacts on WBCs and myeloid cel l disorders lines. Associated with V617F' jAK2 mutation. Polycythemia vera Primary polycythemia. Disorder of t RBCs. May present as intense itching after hot shower. Rare but classic symptom is erythromelalgia (severe, burning pa in and red-blue coloration) due to episodic blood clots in vessels of the extremities a. t EPO {vs 2° polycythemia, which presents with endogenous or artificially t EPO). 6 s 0 lock Suspend End Block 6

0 7 FA17 p 394.1 o8 Pathologic RBC forms

0 9 TYPE ASSOCIATED PATHOLOGY NOTES

0 10 Liver disease, Acantho =spiny.

0 11 ("spur cell") abe tal ipoproteinemia (states of cholesterol dysregulation). 0 12

0 13 0 14 I • 15 Lead poisoning, sideroblastic Seen primarily in peripheral smear, anemi:1s, myclodysplasl'ic vs ringed si deroblasts seen in syndromes. bone marrow. Aggregation of residual ribosomes.

Oacrocyte Bone marrow infiltration (eg, RBC "sheds a tear" because it's ("teardrop cell") B myelofibrosis). mechanical!\' squeezed out of its a s 8 Lock Suspend End Block Item: 7 of 15 ~ 1 • M k -<:J 1>- Jil ~· !:';-~ QIO: 4897 ..L a r Previous Next Lab~lu es Notes Calculat o r

1 • A 45-year-old man presents to the emergency department with his wife, who states that over the past few months he has had increased difficulty 2 walking and seems confused, often forgetting her n ame or putting things In odd places. Today at dinner he started screaming that he saw little men running In his food. Laboratory tests show a mean corpuscular volume of 105 fl. A Schilling test shows a low level of radioactive vitamin 8 12 In his 3 urine 24 hours later; a repeat dose given with intrinsic factor results in high urinary levels of radioactive vitamin 8 12. 4 5 What Is the mechanism of the disease causing this patient's symptoms?

6 : A. Antibody to cobalamin 0 7 o8 B. Antibody to duodenal intrinsic factor-cobalamin receptors

. 9 c. Antibody to ileal intrinsic factor-cobalamin receptors • 10 0. Antibody to intrinsic factor-producing cells • 11 E. Antibody to jejunal intrinsic factor-cobalamin receptors • 12 • 13

0 14 • 15

a s 8 Lock Suspend End Block Item: 7 of 15 ~ 1 • M k -<:J 1>- Jil ~· !:';-~ QIO: 4897 ..L a r Prev ious Next Lab~lu es Notes Calculat or

1 •

2 The correct answ er i s D. 670/o chose this. 3 This patient has pernicious anemia, which is a failure of the body to absorb cobalamin from the diet and usually develops after the age of 40 years. Patients present with symptoms of vitamin 8 12 deficiency, with an insidious onset of neurologic symptoms, including weakness, ataxia, dementia, and 4 psychosis. A complete blood cell count will show a megaloblastic anemia. Cobalamin can be absorbed only when bound to intrinsic factor (IF), through the problem lies in the combination of IF with cobalamin or 5 receptors in the distal ileum. The Schilling test for pernicious anemia will determine whether In the absorption of this complex in the ileum. In this patient, the repeat dose showing high levels of radioactive vitamin 8 12 indicates that he has no 6 problem with absorption. Thus, this patient has type I, or "blocking, • antibody. Megalobla' 1c a1 em1a Pern1c1ous anemia Schilling test Intrinsic factor Ileum Complete blood count Cobalamin Vitamin 812 Ataxia Psychosis Anemia Dement1a 7 8 vitamin Antibody Vitamin Radioactive decay Neurology · 8 A i s not correct. 4 % chose this. . 9 In this patient, the repeat dose in his Schilling test shows high levels of radioactive vitamin 812 in his urine 24 hours later. This indicates that he has no is due to an autoimmune antibody against • 10 problem with absorption, but has a problem with the combination of Intrinsic factor (IF) with cobalamin. This IF-producing cells, which decreases the level of IF, thereby decreasing cobalamin absorption. A direct antibody to cobalamin has not been characterized In • 11 these patients. Schilling test Cobalamin Intrinsic factor Vitamin 8 12 Vitamin Autoimmune disease 8 vitamins Urine Antibody Autoimmunity Radioactive decay • 12 B is not co rrect. 60/o chose this • • 13 This patient does not have a problem in IF-cobalamin absorption. Additionally, receptors for this complex are found only in the distal ileum and would not • 14 be found In the duodenum . Duodenum Ileum Distal Anatomical terms of l ocation Absorption (chemistry) • 15 C is not co rrect . 190/o chose this. The Schilling test indicates that this patient has a problem with IF binding to cobalamin, rather than absorption of the complex. If the repeat dose had also resulted in low radioactivity, then he would have the type II, or "binding," antibody. Schilling test Cobalamin Radioactive decay Antibody

E is not co rrect. 4 0/o chose this. The cause for this patient's pernicious anemia does not lie in intrinsic factor (!F)-cobalamin absorption, but rather is due to the combination of IF with cobalamin. Additionally, receptors for this complex are found only in the distal Ileum, and not in the jejunum. Pernic1ous anemra Jejunum Intrinsic factor Cobalamin Ileum Anemia Anatomica terms of ocation a s 8 Lock Suspend End Block Item: 7 of 15 ~ . I • M k <:] t> al ~· ~ QIO: 4897 .l. ar Previous Next lab 'Vfl1ues Notes Calculator y, yp g, y 1 • • • • • Schilling test Cobalamin Radioactive decay Antibody 2 E is not correct. 4 % chose this. 3 The cause for this patient's pernicious anemia does not lie in intrinsic factor (!F)-cobalamin absorption, but rather is due to the combination of IF with cobalamin. Additionally, receptors for this complex are found only in the distal ileum, and not in the j ejunum. 4 Pernicious anemia Jejunum Intrinsic factor Cobalamin Ileum Anemia Anatomical terms of location 5 6 Bottom Line: 7 Autoimmune antibody against intrinsic factor -producing cells decreases the level of intrinsic factor; thereby decreasing cobalamin absorption. • 8 Cobalamin Intrinsic factor Autoimmune disease Autoimmunity Antibody • 9

0 10 oll lijl;fiiJI•l toryear:[ 2017 • ] FI RST AID FACT S • 12

0 13 FA17 p 398.1 • 14 Macrocytic (MCV > 100 fl) anemia DESCRIPTION FINDINGS 0 15

Megaloblastic anemia Impaired Dl A synthesis -+ maturation of RBC , hypersegmented nucleus of precursor cells in bone marrow neutrophi ls a,g lossitis. delayed rel ative to maturation of cytoplasm .

• 6 s 0 lock Suspend End Block Item: 8 of 15 ~ . I • M k <:] t> al ~· ~ QIO: 1382 .l. ar Previous Next lab 'Vfl1ues Notes Calculator

1 A 22-yea r -old woman presents to her family physician because of increa sing fatigue and because she looks "pale" despite spending many hours lA"' AI 2 outside as a camp counselor. She also states that her urine looks "cola-colored" when she first goes to the bathroom in the morning. The patient feels well otherwise. Blood tests show low levels of RBCs, WBCs, and platelets. 3

4 Which of the following best describes the pathophysiology behind this patient's most likely disorder? 5 : 6 7 A. Complement-mediated hemolysis of RBC due to loss of glycosylphosphatidylinositol (GPI)-anchored proteins CD59 and CDSS on the surface of . 8 RBCs • 9 B. Defect in proteins interacting with the RBC membra ne cytoskeleton and plasma membrane, such as ankyrin, band 3, protein 4.2, and spectrin

• 10 c. IgG-mediated RBC hemolysis · 11 D. IgM -mediated RBC hemolysis • 12 E. Point mutation lea ding to a single amino acid replacement in the J3-chain of hemoglobin causing hemolysis • 13 • 14 F. Reduction in the half-life of glucose-6-phosphate dehydrogenase lea ding to decrea sed glutathione and increa sed RBC susceptibility to oxidant stress • 15

6 s 0 lock Suspend End Block Item: 8 of 15 ~ . I • M k <:] t> al ~· ~ QIO: 1382 .l. ar Previous Next lab 'Vfl1ues Notes Calculator

1 The correct a nswer is A. 72% chose this. 2 This patient presents with signs and symptoms of anemia. The dark urine is due to the presence of hemoglobin, which occurs only in the setting of 3 intravascular hemolysis. Urine color change points to paroxysmal nocturnal hemoglobinuria (PN H), a rare form of hemolytic anemia. Hemolysis in PN H occurs throughout the day, but at night, urine becomes concentrated, and this lea ds to the dramatic change in urine color. We would expect lab tests to 4 show a normocytic anemia, elevated unconjugated bilirubin and lactate dehydrogenase (LDH) levels, and low haptoglobin. PN H classically manifests as 5 the triad of hemolytic anemia, pancytopenia, and thrombosis. In PN H, a defective protein known as glycosylphosphatidylinositol (GPI) anchor (encoded by the PIG-A gene) is present on the RBC membrane. Normally, GPI attaches proteins to the surface of RBCs and prevents the attachment of complement to 6 the membrane and subsequent lysis. Flow cytometry results demonstrate CDSS/ 59 negative RBCs. Trea tment is with eculizumab, a terminal complement inhibitor. I 7 Paroxysmal nocturnal hemoglobinuria Bilirubin Flow cytometry Pancytopenia Eculizumab Hemoglobin lactate dehydrogenase Haptoglobin Gene Hemolysis 8 Hemolytic anemia Anemia Hemoglobinuria Glycophosphatidylinositol Urine Protein lysis Thrombosis Normocytic anemia Red blood cell lactic acid Blood vessel

• 9 B is not correct. 5 % chose this. • 10 This is the mechanism of action for hereditary spherocytosis, a primarily intrinsic hemolytic anemia. Loss of these proteins lea ds to formation of spherocytes, small round RBC with increa sed MCHC and premature removal by the spleen. Patients present with splenomegaly and aplastic crisis after · 11 infections (parvovirus B19). Diagnosis is by a positive osmotic fragility test. Trea tment is with splenectomy. Hereditary spherocytosis Parvovirus B19 Splenectomy Splenomegaly Hemolytic anemia Anemia Erythrocyte fragility Spleen Spherocytosis Hemolysis Parvovirus • 12 Mean corpuscular hemoglobin concentration Red blood cell Reticulocytopenia • 13 c is not correct. 8 % chose this • • 14 This is the mechanism of action for warm autoimmune hemolytic anemia, a primarily extrinsic hemolytic anemia. This is a chronic anemia seen in SLE and • 15 CLL and as a result of some drugs, such as a-methyldopa. Usually Coombs positive . Warm autoimmune hemolytic anemia Hemolytic anemia Autoimmune hemolytic anemia Autoimmune disease Systemic lupus erythematosus Hemolysis Autoimmunity Anemia Mechanism of action leukemia

D is not correct. 5 % chose this. This is the mechanism of action for cold autoimmune hemolytic anemia. In the cold, IgM binds to RBCs and fixes complement lea ding to primarily extrinsic hemolysis. This condition is associated with CLL, Mycoplasma pneumoniae, and infectious Mononucleosis. RBC agglutinates can lea d to painful, blue fingers and toes when exposed to the cold. Usually a Coombs-positive anemia. Mycoplasma pneumoniae Autoimmune hemolytic anemia Hemolysis Infectious mononucleosis Hemolytic anemia Immunoglobulin M Autoimmunity Agglutination (biology) Autoimmune disease Anemia Mycoplasma Cold autoimmune hemolytic anemia Red blood cell B-cell chronic lymphocytic leukemia 6 s 0 lock Suspend End Block Item: 8 of 15 ~ . I • M k <:] t> al ~· ~ QIO: 1382 .l. ar Previous Next lab 'Vfl1ues Notes Calculator

1 D is not correct. 5 % chose this. 2 This is the mechanism of action for cold autoimmune hemolytic anemia. In the cold, IgM binds to RBCs and fixes complement lea ding to primarily extrinsic hemolysis. This condition is associated with CLL, Mycoplasma pneumoniae, and infectious Mononucleosis. RBC agglutinates can lea d to painful, blue 3 fingers and toes when exposed to the cold. Usually a Coombs-positive anemia. Mycoplasma pneumoniae Autoimmune hemolytic anemia Hemolysis Infectious mononucleosis Hemolytic anemia Immunoglobulin M Autoimmunity 4 Agglutination (biology) Autoimmune disease Anemia Mycoplasma Cold autoimmune hemolytic anemia Red blood cell B-cell chronic lymphocytic leukemia 5 E is not correct. 3 % chose this. 6 This is the mechanism of action for sickle cell anemia, a primarily intrinsic hemolytic anemia. In sickle cell anemia, there is a substitution of glutamic acid 7 for valine on the j3 -chain of hemoglobin. Patients present with numerous complications, including aplastic crises, autosplenectomy, painful crises, Salmonella osteomyelitis, and renal papillary necrosis. Symptoms begin in childhood. Diagnosis of this condition is through hemoglobin electrophoresis. 8 Trea tment is with hydroxyurea, which increa ses HbF, and hydration. Renal papillary necrosis Osteomyelitis Hydroxycarbamide Autosplenectomy Sickle-cell disease Hemoglobin electrophoresis Valine Hemoglobin Glutamic acid I • 9 Hemolytic anemia Anemia Salmonella Necrosis Fetal hemoglobin Sickle Hemolysis Kidney Electrophoresis • 10 F is not correct. 7 % chose this. · 11 This is the mechanism of action of G6PD deficiency, a primarily intrinsic hemolytic anemia. Hemolysis usually occurs following oxidant stress (sulfa drugs, • 12 antimalarials, infections, fava bea ns). Patients present with hemoglobinuria a few days after an oxidant stress. Blood smea r shows Heinz bodies and bite cells. • 13 Heinz body Glucose-6-phosphate dehydrogenase deficiency Hemolysis Sulfonamide (medicine) Hemolytic anemia Degmacyte Anemia • 14 Glucose-6-phosphate dehydrogenase Oxidative stress Vicia faba Blood film Antimalarial medication Malaria Hemoglobinuria Oxidizing agent • 15

Botto m Line: Paroxysmal nocturnal hemoglobinuria is a disorder in the synthesis of GPI anchor proteins. It is characterized by intrava scular hemolysis, pancytopenia, and venous thromboses in atypical sites. Hemolysis in PNH occurs throughout the day, but urine becomes concentrated at night, and this lea ds to the dramatic change in color. Paroxysmal nocturnal hemoglobinuria Pancytopenia Hemolysis Hemoglobinuria Glycophosphatidylinositol Hemolytic anemia Urine Thrombosis Venous thrombosis Blood vessel Protein

6 s 0 lock Suspend End Block Item: 8 of 15 ~ . I • M k <:] t> al ~· ~ QIO: 1382 .l. ar Previous Next lab 'Vfl1ues Notes Calculator

1 Glucose-6-phosphate dehydrogenase Oxidative stress Vicia faba Blood film Antimalarial medication Malaria Hemoglobinuria Oxidizing agent

2 3 Bottom Line: 4 Paroxysmal nocturnal hemoglobinuria is a disorder in the synthesis of GPI anchor proteins. It is characterized by intrava scular hemolysis, pancytopenia, and venous thromboses in atypical sites. Hemolysis in PNH occurs throughout the day, but urine becomes concentrated at night, and this leads to the 5 dramatic change in color. Paroxysmal nocturnal hemoglobinuria Pancytopenia Hemolysis Hemoglobinuria Glycophosphatidylinositol Hemolytic anemia Urine Thrombosis Venous thrombosis 6 Blood vessel Protein 7

8 • 9 lijl;fiiJI•l toryear:[2017 • ] FIRST AID FACTS • 10 · 11 FA17 p400.1 • 12 Intrinsic hemolytic anemia • 13 DESCRIPTION FINDINGS • 14 Hereditary Extravascular hemolysis due to defect in Splenomegaly, aplastic crisis (parvov irus Bl9 • 15 spherocytosis proteins interac ting with RBC membrane infectio n). skeleton and plasma membrane (eg, ankyrin, Labs: osmotic fragility test®. 1 onnal to band 3, protein 4.2, spectrin). Mostly l MCV wi th abundance of cells. autosomal dominant inheritance. Treatment: splenectomy. Results in small, round RBCs with less surface area and no central pa llor (t MCI !C) - premature removal by spleen. G6PD deficiency Most common enzymatic disorder of RBCs. Back pain, hemoglobinuria a few days after 6 s 0 lock Suspend End Block 3 HbC disease Glutamic acid-to-lyC ine (I) sine) mutation in Pa tients with llbSC (I of each mutant gene) have ~ -g l obin. Causes extravascular hemol}sis. milder disease than llbSS patients. 4 Blood smear in homozrgotes: hemoglobin 5 C rr~tals inside RBCs, target cells. 6

7 FA17 p 561 .2 8 Acidosis and alkalosis . 9 Check arterial pH • 10

• 11 pH< 7.35 pH> 745 • 12 Acidemia Alkalemia • 13 Pco > 44 mm Hg Pco <36 mm Hg > 28 mEq/L • 14 2 2 .GKo~- • 15 I Respiratory Respiratory Metabolic acidosis Metabolic alkalosis acidosis alkalosis I l gap Hypoventit.tion Check anion Hyperventit.tion H• loss/HC05- excess = Na•- (Ct- t HCO; l A1rway obstruction Hysteria loop diuretics Acute lung disease Hypoxemia (eg. high attitude) Vomiting Chronic tung disease Salicylates ea( rly) Antacid use Opioids, sedatives Tumor Hyperaldosteromsm a s 8 Lock Suspend End Block Item: 8 of 15 ~ 1 • M k -<:J 1>- Jil ~· !:';-~ QIO: 1382 ..L a r Previous Next Lab~lu es Notes Calculat o r

1 pH •

2

3 FA17 p635.1 4 Carbon dioxide CO, is transported from tissues to lungs in 3 In lungs, oxygenation of l-I b promotes 5 transport forms: dissociation of H+ from Hb. This shifts 6 0 HC03- (70%). equilibrium to"·ard C02 formation; therefore, f) Carbaminohemoglobin or HbC0 C0 is released from RBCs (Haldane effect). 7 2 2 (21-2;%). C02 bound to lib at 1 -terminus In peripheral tissue, t H+ from tissue 8 of globin (not heme). C02 binding favors metabolism shifts curve to right, unloading 0 2 . 9 taut form (0 2 unloaded). (Bohr effect). • 10 0 Dissolved C02 (5-9%}. 1\ lajority of blood C02 is carried as HC03- in • 11 the plasma .

• 12 ~- __C....._ l- / HCO,- 0 • 13 Tissue Plasma RBC • 14 • 15 C02 enters RBC and is converted to HCO,- rbom r 1ydra HC0 ~-~ H' + HCO,- C02 + H,O 2 1 HHb ~-- H' + Hb- I CO_t Hb HbCO_Q a s 8 Lock Suspend End Block Item: 9 of 15 ~ 1 • M k -<:J 1>- Jil ~· !:';-~ QIO: 3459 ..L a r Previous Next Lab~lu es Notes Calculat o r

1 • A 36-year-old white woman with a history of uncomplicated systemic lupus erythematosus presents to her physician with edema and pain in her right lA• A] 2 foot that began 2 days ago. Physical examination reveals a positive Homan sign (pain on dorsiflexion of the foot), and an ultrasound reveals a nonoccluslve thrombus in the right popliteal vein. Laboratory studies show a platelet count of 100,000/mm•, a prothrombin time of 24 seconds, and 3 partial thromboplastin time of 89 seconds. 4 5 What Is the most likely cause of this patient's condition?

6 : A. AntibOdies directed against factor VIII 7 8 B. AntibodieS directed against heparin

. 9 c. Antibodies directed against platelet glycoprotein lib/ Ilia • 10 o. Antibodies directed against platelet phospholipids • 11 E. Antibodies directed against RBCs • 12 • 13 • 14 • 15

a s 8 Lock Suspend End Block Item: 9 of 15 ~ 1 • M k -<:J 1>- Jil ~· !:';-~ QIO: 3459 ..L a r Prev ious Next Lab~lu es Notes Calculat or

1 •

2 The correct answ er i s D. 580/o chose this. 3 This patient has developed a lupus anticoagulant (a type of antiphosphollpld antibody), and shows signs and symptoms typical of antiphosphollpld antibody syndrome. Patients with antiphospholipid antibody syndrome have prolongation of coagulation times in vitro, but a paradoxical increased 4 likelihood of forming clots in vivo. Platelet phospholipids are required for both the intrinsic and extrinsic clotting pathways. Antiphospholipid antibodies leading to recurrent venous and arterial thrombosis. Because the 5 bind to platelet phospholipids, thereby making them accessible to clotting factors and partial thromboplastin time (PTT) assays use exogenous phospholipids, the antibOdies inhibit their function and paradoxically show an increase In 6 coagulation t1me. While antiphospholipid syndrome often presents with a normal platelet count, this patient has systemic lupus erythematosus, which Is often associated with thrombocytopenia. Lupus anticoagulant is also associated with recurrent miscarriage. 7 Lup10s antico<~gulant Anticoagulant Systemic lupus erythematosus ~rombocytopenoa Partia thromboplastin time Platelet Antiphospholipid syndrome Antibody 8 '"hrombos·s Lupus erythematosus Phospholipid In vitro Recurrent miscarriage In vivo Coagulation Thromboplastin Miscarriage Platelet count vein 9 A is not correct. 13% chose this. is a rare cause of acquired hemophilia In • 10 Systemic lupus erythematosus (S LE ) can be associated with generation of factor VIII autoantibodies, which adults. Their presence would cause bleeding rather than thrombosis seen In this case. Since factor VIII is part of the intrinsic cascade, we would expect an • 11 Increase In the partial thromboplastin time and a normal prothrombin time . Systemic lupus erythematosus Prothrombin time Haemophilia Partial thromboplastin time Factor VIII Thrombin Autoantibody Thrombosis Lupus erythematosus • 12 Thromboplastin • 13 8 is not co rrect. 11 Ofo chose this• • 14 Heparin-Induced thrombocytopenia is a hypercoagulable state caused by an Immune reaction to exogenous heparin. This patient has no history of heparin • 15 exposure . Heparin-induced thrombocytopenia Thrombocytopenia Heparin Thrombophilia Immune system Coagulation

C is not co rrect. 140/o chose this. Antibodies directed against platelet glycoproteins, especially lib/Ilia, lead to Idiopathic thrombocytopenic purpura. This is characterized by extremely low platelet levels but normal prothrombin and partial thromboplastin times. Immune thrombocytopenic purpura Thrombin Antibody Platelet Glycoprotein Purpura Idiopathy

E is not correct. 4 0/o chose this. Antibodies directed against red blood cell antigens lead to hemolytic anemia. They are unrelated to clotting disorders. Red blood eel Hemolytic anemia Anemia Antibody Antigen Hemolysis Coagulatoon Blood ce a s 8 Lock Suspend End Block Item: 9 of 15 ~ . I • M k <:] t> al ~· ~ QIO: 3459 .l. ar Previous Next lab 'Vfl1ues Notes Calculator

1 Antibodies directed against platelet glycoproteins, especially lib/ Ilia, lead to idiopathic thrombocytopenic purpura. This is characterized by extremely low platelet levels but normal prothrombin and partial thromboplastin times. 2 Immune thrombocytopenic purpura Thrombin Antibody Platelet Glycoprotein Purpura Idiopathy

3 E is not correct. 4 % chose this. 4 Antibodies directed against red blood cell antigens lead to hemolytic anemia. They are unrelated to clotting disorders. Red blood cell Hemolytic anemia Anemia Antibody Antigen Hemolysis Coagulation Blood cell 5 6 Bottom Line: 7 Antiphospholipid antibody syndrome, or lupus anticoagulant, is a result of antibodies directed against platelet phospholipids, causing an increase in 8 coagulation time. It is associated with recurrent thrombosis and miscarriages. 9 lupus anticoagulant Anticoagulant Antiphospholipid syndrome Platelet Antibody lupus erythematosus Coagulation Phospholipid Thrombosis Systemic lupus erythematosus

0 10 oll

0 12 Iill ;fi 1!1J•J for year: 2017 " FI RST AI D FACTS 0 13

0 14 FA17 p443.1 0 15 Systemic lupus erythematosus SYMPTOMS Classic presentation: rash, joint pain, and fever, RASH OR PAIN: most commonly in a female of reproductive Rash (malar rJ or discoid 1]1) age and African-American descent. Arthritis (nonerosive) Li bman-Sacks Endocarditis-nonbacterial, Serositis verrucous th rombi usua lly on mitral or aortic Hematologic disorders (eg, cytopenias) va lve and can be present on either surface of Oral/nasopharyngeal ulcers thP v::~ l vP f h 11 i· 11 < 11 <:~ l l v nn llnrlP rrP\ fT 5 ;II. R Pn::~ l rl ioP' !!":-~ Ite m: of ~ 1 a r -<:J I> fJ • QIO: 3459 ..L Previous Next Lab lues Not es Calculator • • • 1

2 FA17 p 392.1 3 Coagulation cascade components 4 Procoagulation Warfarin inhibits the enzvme vitamin K 5 epoxide reductase. 6 epollde laos as c: inadive II, VII. IX. X. C. S 'eonates lack enteric bacteria, which produce OXIdiZed _red_ut_u_~._ reduced _col_~_t_Of_l - ~U~t~n.,...., 7 v1tam1n K vrtarrun K vitamin K. 8 mature (actrve) 11. VII. IX. X. C. S Vitamin Kdeficiency: I s~ nthesis of factors II, Vl l, protein S. 9 lX, X, protein C, '\\ F carries/protects \'HI (' olks\\'agen • 10 Factories make gr8 (great) cars . • 11 Anticoagulation Antithrombin inhibits activated forms offactors • 12 thromb1n thrombomodubn complex ll, VlT, IX, X, XI, Xli. • 13 !endothelial cells} prolm S l leparin enhances the activity of antithrombin . Protein C activated protein C - cleaves and inactivates Va, VIlla • 14 Principal targets of antithrombin: thrombin and • 15 factor Xa . tPA Plasminogen - - plasmin - - fibrinolysis: Factor V Leiden mutation produces a factor V 1. cleavage or fibrin mesh resistant to inhibition by activated protein C. 2. destruction of coagulation Iactors tPA is used clinically as a thrombolytic.

FA17 p 393.2 Thrombogenesis Formation of insoluble fibrin mesh. A~nirin irrt>vt>rn~~P a s 8 Lock Suspend End Block Item: 10 of 15 ~ 1 • M k -<:J 1>- Jil ~· !:';-~ QIO: 4545 ..L a r Previous Next Lab~lu es Notes Calculat o r

1 • A young woman is admitted to the hospital after being paralyzed from the waist down In a motor vehicle crash. It is determined that she does not 2 have Intracranial bleeding, and she is placed on subcutaneous heparin for deep venous thrombosis prophylaxis as well as oxycodone and acetaminophen as needed for pain. Her platelet levels during her admission are: 3 Days 1-4: 200,000/mm• 4 Day 5: 150,000/mm• 5 Day 6: 80,000/mm• Day 7: 76,000/mm• 6 7 Besides an Increased risk of bleeding, what else is this woman at most Immediate risk for if she continues on her cu rrent treatment regimen? 8 9 : A. Ascending paralysis • 10

• 11 B. Intravascular thrombosis

• 12 c. Jaundice • 13 D. Rhabdomyolysis • 14 E. Skin necrosis • 15

a s 8 Lock Suspend End Block Item: 10 of 15 ~ 1 • M k -<:J 1>- Jil ~· !:';-~ QIO: 4545 ..L a r Prev ious Next Lab~lu es Notes Calculat or

1 •

2 The correct answ er i s B. 530/o chose this. This patient most likely has thrombocytopenia secondary to heparin administration. Heparin-induced thrombocytopenia (HIT) is an immunologic reaction 3 to heparin, producing antibodies that cross-rea ct with platelet factor 4, leading to platelet activation and clumping. HIT type 1 arises within a few days of 4 treatment and is not an antibody-mediated response. Platelet levels usually do not drop below 100,000/ mm•. HIT type 2 is due to new formation of heparin antibodies and usually manifests 4-5 days after starting heparin therapy. Platelet levels typically drop below 100,000/ mm3. Because of the 5 activation and clump111g of platelets in these patients, heparin must be stopped Immediately and a nonheparin anticoagulant (ie, argatroban) must be used to prevent arterial thromboses. Thromboses may be venous and/or arterial. 6 Hepa in- rduced thrombocttopenia Platelet factor 4 Anticoagulant ~rombocytopenia A

8 A i s not correct. 3% chose this. 9 Ascending paralysis is an unlikely result of HIT and would not be the most proximal concern of the medical team. Paralysis Ascending paralysis Anatomical terms of location 10 C is not correct. 16% chose this• • 11 Although jaundice could result from thromboses of vessels of the liver, It Is not the most immediate risk of HIT. • 12 Jaundice Uver Thrombosis • 13 0 is not co rrect. 90/o chose this • Rhabdomyolysis is not associated with HIT, and her trea tment regime does not place her at a significantly increa sed risk for rhabdomyolysls. • 14 Rhabdomyolysis • 15 E is not co rrect . 190/o chose this. Skin necrosis Is typically associated with warfarin use, an anticoagulant that antagonizes vitamin K and therefore inhibits the production of vitamin K­ dependent clotting factors. Skin necrosis is not typically associated with HIT. Warfarin Anticoagulant Vitamin K Necrosis Coagulation Warfarin necrosis Vitamin

Bottom Line: Heparin-Induced thrombocytopenia (HIT) is an immunologic reaction to heparin. HIT type 1 is not an antibody-mediated response, and platelet levels usually do not drop below 100,000/ mm•. HIT type 2 is due to new formation of heparin antibodies, and platelet levels typically drop below ...... AAAI-•-• • ~~-•--l. •••• -•••• l. . • • • - • • • • · - --· - ' ·' ·- '-·"'"'- a s 8 Lock Suspend End Block Item: 10 of 15 ~ . I • M k <:] t> al ~· ~ QIO: 4545 .l. ar Previous Next lab 'Vfl1ues Notes Calculator . . y y . - . g . p g y . . Y' y 1 Rhabdomyolysis 2 E is not correct. 19% chose this. 3 Skin necrosis is typically associated with warfarin use, an anticoagulant that antagonizes vitamin K and therefore inhibits the production of vitamin K­ dependent clotting factors. Skin necrosis is not typically associated with HIT. 4 Warfarin Anticoagulant Vitamin K Necrosis Coagulation Warfarin necrosis Vitamin 5 6 Bottom Line: 7 Heparin-induced thrombocytopenia (HIT) is an immunologic reaction to heparin. HIT type 1 is not an antibody-mediated response, and platelet levels 8 usually do not drop below 100,000/mm• . HIT type 2 is due to new formation of heparin antibodies, and platelet levels typically drop below 100,000/mm• . Thromboses may be venous or arterial or both. 9 Heparin-induced thrombocytopenia Thrombocytopenia Platelet Heparin Antibody Immune system Thrombosis Vein 10 · 11 • 12 lijl;fiiJI•l toryear:[2017 • ] FI RST AI D FA CTS • 13 • 14 FA17 p413.1 • 15 Heparin MECHANISM Lowers the activity of thrombin and factor Xa. Short half-life.

CLINICAL USE Immediate anticoagulation for pulmonary embolism (PE), acute coronary sy ndrome, M I, deep venous thrombosis (OVT}. Used during pregnancy (does not cross placenta). Follow PTr. ADVERSE EFFECTS Bleeding, thrombocytopenia (HIT), osteoporosis, drug-drug interactions. For rapid reversal (antidote), use protamine sulfate (positi vely charged molecule that binds negatively charged heparin}. 6 s 0 lock Suspend End Block Item: 11 of 15 ~ 1 • M k -<:J 1>- Jil ~· !:';-~ QIO: 4478 ..L a r Previous Next Lab~lu es Notes Calculat o r

1 • A 9-month-old girl is found to be anemic. Her mother and several great aunts have a similar history. Results of a peripheral blood smear suggest 2 thalassemia major.

3 Mean Mean Hemoglobin Retleuloeyle corpuscular Serum Iron 4 Cholee corpuscular (gldl) count(%) hemoglobin (fJg/dl) (Umln) volume (IL) 5 6 A 5.5 6 18 60 100 7 - f- -- - . 8 B 7.5 3 26 70 30 9 10 c 8.0 I 27 110 75 • 11 • 12 0 9.0 2 31 92 225 • 13 -1- • 14 E 10.5 2 30 91 40 • 15

What laboratory values (shown in the table above) are most likely to be seen In this patient?

A

B c a s 8 Lock Suspend End Block Item: 11 of 15 ~ 1 • M k -<:J 1>- Jil ~· !:';-~ QIO: 4478 ..L a r Previous Next Lab~lu es Notes Calculat o r • 1 •

2 A 5.5 6 18 60 100 3 - - -- -· - 4 B 7.5 3 26 70 30 5 6 c 8.0 1 27 110 75 7

8 D 9.0 2 31 92 225 9 - f- 10 E 10.5 2 30 91 40

• 11 • 12 What laboratory values (shown in the table above) are most likely to be seen In this patient? • 13 : • 14 A • 15 B c

0

E

a s 8 Lock Suspend End Block Item: 11 of 15 ~ 1 • M k -<:J 1>- Jil ~· !:';-~ QIO: 4478 ..L a r Prev ious Next Lab~lu es Notes Calculat or

1 •

2 The c orrect answ er i s A. 600/o chose this. In thalassemia major, hypochromic, microcytic anemia is seen, and the reticulocyte count is often elevated 3 due to Increased production of RBCs by the bone marrow. Importantly, the serum Iron is usually normal or can distinguish thalassemia major from Iron deficiency anemia. A 4 high, which is another indicator test that blood smear from a patient with thalassemia major resembles that in the Image. 5 Reticulocyte Mic ocytic anem1a Iron-deficiency anemia Thalassemia Serum ron Blood fo m 6 Mean corpuscular hemoglobin concentration Iron deficiency Anemia Bone marrow Beta thalassemia Blood plasma Serum (blood) Iron 7 I 8 9 10

11 • 12 B is not co rrect. 230/o chose this • • 13 These Indices are consistent with iron-deficiency anemia. The reticulocyte count Is a measure of how fast immature RBCs are being produced by the bone • 14 marrow. It Is reduced in the setting of iron deficiency because there Is Insufficient Iron for RBC synthesis. Serum iron is also reduced, indicating a deficiency In Iron in this patient. Another lab value, total iron-binding capacity {TIBC), will be increased in iron deficiency anemia; however, that Is not • 15 tested In this vignette . Reticulocyte Iron-deficiency anemia Serum iron Total iron-binding capacity Iron deficiency Anemia Bone marrow Blood plasma Serum (blood) Red blood cell Iron

C is not co rrect . 60fo chose this. The MCV In this set of lab values is 110, which is diagnostic of a macrocytic anemia. The patient in the vignette has thalassemia, which is characterized by a microcytic anemia. Microcytic anemia Macrocytic anemia Thalassemia Macrocytosis Mean corpuscular volume Anemia

0 is not correct. 8 0/o chose this. The MCHC and MCV in this set of lab va lues reveal a normochromic, normocytic anemia, which is inconsistent with this patient's diagnosis of thalassemia. The Increased iron level suggests a diagnosis of sideroblastic anemia. Slderoblastlc anemia often presents with - but the MCV can be normal a s 8 Lock Suspend End Block Item: 11 of 15 ~ . I • M k <:] t> al ~· ~ QIO: 4478 .l. ar Previous Next lab 'Vfl1ues Notes Calculator

1 The MCV in this set of lab values is 110, which is diagnostic of a macrocytic anemia. The patient in the vignette has thalassemia, which is characterized by a microcytic anemia. 2 Microcytic anemia Macrocytic anemia Thalassemia Macrocytosis Mean corpuscular volume Anemia 3 D is not correct. 8 % chose this. 4 The MCHC and MCV in this set of lab values reveal a normochromic, normocytic anemia, which is inconsistent with this patient's diagnosis of thalassemia. The increased iron level suggests a diagnosis of sideroblastic anemia. Sideroblastic anemia often presents with microcytosis - but the MCV can be normal 5 or even high. 6 Thalassemia Normocytic anemia Anemia Sideroblastic anemia Microcytosis Mean corpuscular hemoglobin concentration Normochromic anemia 7 Mean corpuscular volume Iron 8 E is not correct. 3 % chose this. The MCHC and MCV in this set of lab values reveal a normochromic, normocytic anemia. Together with the low serum iron, these indices suggest anemia 9 of chronic disease. The MCV in anemia of chronic disease is usually normal but can be slightly decreased. Anemia of chronic disease Anemia Normochromic anemia Mean corpuscular hemoglobin concentration Serum iron Normocytic anemia Mean corpuscular volume 10 Blood plasma Chronic condition Iron 11 • 12 Botto m Line: • 13 Thalassemia major presents with hypochromic, microcytic anemia; elevated reticulocyte count; and normal serum iron measurements. • 14 Reticulocyte Beta thalassemia Microcytic anemia Thalassemia Anemia Serum iron Mean corpuscular hemoglobin concentration Blood plasma Hypochromic anemia • 15 Iron Serum (blood)

liU ;fi 1!1J t) fo r year:[ 2017 .. ] FI RST AID FACTS

FA17 p 396.2 Microcytic (MCV < 80 fl), hypochromic anemia 6 s 0 lock Suspend End Block 2

3 FA17 p 396.2 4 Microcytic (MCV < 80 fl), hypochromic anemia 5 Iron deficiency l iron due to chronic bleeding (eg, Gl loss, menorrhagia), malnutrition, absorption disorders, Gl 6 surgery (eg, gastrectomy), or t demand (eg, pregnancy) -+ l final step in heme synthesis. Labs: l iron, t TTBC, l ferritin, t free er} I hrOC) te protoporphyrin. \l(icrocytosis and 7 h} pochromasia (central pallor) 8 Symptoms: fatigue, conjuncti,·al pallor , pica (consumption of nonfood substances), spoon nails 9 (koilonychia). 10 !\lay manifest as glossitis, cheilosis, Plummer-Vinson syndrome (triad of iron deficiency anemia, 11 esophageal webs, and dysphagia). • 12 a -thalassemia Defect: a-globin gene deletions- l a -globin sy nthesis. cis deletion (both deletions occur on • 13 same chromosome) prevalent in Asian populations; trans deletion (deletions occur on separate chromosomes) pre"a lent in African populal ions . • 14 4 allele deletion: no a-globin. Excess y-glo!Jin fonn s y4 (I IIJ Barts). Incompatible with life (causes • 15 hydrops fetalis). 3 allele deletion: inheritance of chromosome with cis deletion+ a chromosome with I allele deleted - HbH disease. Very litt le a -globin. Excess ~ -g lobin forms ~4 (HbH). I 2 allele deletion: less clinically severe anemia. I allele deletion: no anemia (clinically silent).

~ -tha l a sse mia Point mutations in spl ice sites and promoter sequences - l ~-gl obin synthesis. Prevalent in Mediterranean populations. R...th::al::tc:c:omi::a minnr f hl'l-fPrl\?\·nnt/lo\• R ~h·~t,, ~ ~ ••nriPrnrnAn,...I'-A I l('u~llu "lC'"n "'nfnnv:l.;l" IJi•:tonnc-ic a s 8 Lock Suspend End Block Item: 12 of 15 ~ 1 • M k -<:J 1>- Jil ~· !:';-~ QIO: 3857 ..L a r Previous Next Lab~lu es Notes Calculat o r

1 • An 18-year-old woman presents with a "bleeding problem." Coagulation studies are notable for a prolonged bleeding time, prolonged partial IA•A] 2 thromboplastin time, decreased factor VIII levels, and a cofactor assay failed to show increased platelet aggregation. Results of complete blood count are as follows: 3 4 Hemoglobin: 11.0 g/ dl Hematocrit: 33o/o 5 Leukocyte count: 5400/ mm3 6 Platelet count: 200,000/ mm• 7 8 Which of the following is most likely to have been the initial presentation of the "bleed1ng problem"?

9 : A. Diffuse petechiae and purpura 10

11 B. Excessive bleeding after extraction of an impacted molar 12 • c. Hepatosplenomegaly • 13 0. Knee pain and swelling after light trauma against a table • 14 • 15 e. Schistocytes on peripheral blood smea r

a s 8 Lock Suspend End Block Item: 12 of 15 ~ . I • M k <:] t> al ~· ~ QIO: 3857 .l. ar Previous Next lab 'Vfl1ues Notes Calculator

1

2 The correct a nswer is B. 54% chose this. 3 The laboratory results indicate a likely diagnosis of von Willebrand disea se (vWD), an autosomal dominant dysfunction of von Willebrand factor (vWF) that results in impaired adhesion of platelets to collagen. Mucocutaneous bleeding, which may occur after dental extraction, is the most common initial 4 presentation. In patients with moderate disea se, hemarthroses are ra re and generally occur only with major trauma. Instea d, hemarthrosis is associated more commonly with hemophilia A or B. Impaired platelet function produces a prolonged bleeding time, which assesses primary hemostasis. vW F is a 5 carrier molecule for factor VIII, extending its half-life. Thus vWD results in decrea sed factor VIII levels and a prolonged partial thromboplastin time. Reduced ristocetin-induced platelet aggregation, a test in which ristocetin, formerly used as an antibiotic, stimulates platelets to aggregate, is a specific 6 and sensitive indication of vWD. 7 Hemarthrosis Von Willebrand disease Von Willebrand factor Haemophilia Ristocetin Hemostasis Factor VIII Partial thromboplastin time Collagen

8 Dominance (genetics) Haemophilia A Platelet Antibiotics Autosome Bleeding time Half-life Mucocutaneous zone Platelet aggregation Major trauma 9 A is not correct. 22% chose this. Petechiae and purpura are common signs of thrombocytopenia, which may be a result of aplastic anemia, splenic sequestration, disseminated 10 intrava scular coagulation, thrombotic thrombocytopenic purpura, idiopathic thrombocytopenic purpura, or medications, among others. La boratory findings would include thrombocytopenia, which is not seen in this case. 11 Disseminated intravascular coagulation Thrombotic thrombocytopenic purpura Immune thrombocytopenic purpura Aplastic anemia Petechia Thrombocytopenia 12 Anemia Purpura Coagulation Idiopathy Spleen Thrombosis Blood vessel

• 13 c is not correct. 3 % chose this. • 14 Hepatosplenomegaly may be an indication of a number of conditions, including hepatic dysfunction. This would result in a generalized clotting factor deficiency that manifests as a bleeding disorder. However; patients with this condition would not demonstrate a reduced ristocetin-induced platelet • 15 aggregation, making this answer less likely. Hepatosplenomegaly Platelet Coagulopathy Coagulation liver Von Willebrand disease Platelet aggregation liver disease

D is not correct. 17% chose this. Hemarthroses are common in coagulation factor defects such as hemophilia A and B, involving factors VIII and IX, respectively. Hemophilia A and B are genetically inherited disorders that follow an X-linked recessive transmission pattern, making it very unlikely for female patients to exhibit the disea ses. Both types characteristically include a prolonged partial thromboplastin time and a normal prothrombin time. Pa tients with hemophilia have a normal bleeding time, as this test mea sures primary hemostasis, a process that does not involve the coagulation cascade and simply quantifies the functionality of platelets. In these disea ses, a reduced ristocetin-induced platelet aggregation would not be a laboratory finding. Prothrombin time Haemophilia Hemarthrosis Haemophilia A Partial thromboplastin time Hemostasis Platelet Coagulation Thrombin Bleeding time Recessive Factor VIII X-linked recessive inheritance Dominance laenetics\ Sex linkaae Platelet aaareaation Genetic disorder 6 s 0 lock Suspend End Block Item: 12 of 15 ~ . I • M k <:] t> al ~· ~ QIO: 3857 .l. ar Previous Next lab 'Vfl1ues Notes Calculator

1 Hepatosplenomegaly may be an indication of a number of conditions, including hepatic dysfunction. This would result in a generalized clotting factor deficiency that manifests as a bleeding disorder. However; patients with this condition would not demonstrate a reduced ristocetin-induced platelet 2 aggregation, making this answer less likely. 3 Hepatosplenomegaly Platelet Coagulopathy Coagulation liver Von Willebrand disease Platelet aggregation liver disease 4 D is not correct. 17% chose this. Hemarthroses are common in coagulation factor defects such as hemophilia A and B, involving factors VIII and IX, respectively. Hemophilia A and B are 5 genetically inherited disorders that follow an X-linked recessive transmission pattern, making it very unlikely for female patients to exhibit the disea ses. Both types characteristically include a prolonged partial thromboplastin time and a normal prothrombin time. Pa tients with hemophilia have a normal 6 bleeding time, as this test mea sures primary hemostasis, a process that does not involve the coagulation cascade and simply quantifies the functionality 7 of platelets. In these disea ses, a reduced ristocetin-induced platelet aggregation would not be a laboratory finding. Prothrombin time Haemophilia Hemarthrosis Haemophilia A Partial thromboplastin time Hemostasis Platelet Coagulation Thrombin Bleeding time Recessive 8 Factor VIII X-linked recessive inheritance Dominance (genetics) Sex linkage Platelet aggregation Genetic disorder 9 E is not correct. 4 % chose this. 10 Schistocytes are an indication of intrava scular hemolysis and can be present in conditions that also include excessive bleeding, such as disseminated intrava scular coagulation and thrombotic thrombocytopenic purpura. 11 Disseminated intravascular coagulation Thrombotic thrombocytopenic purpura Hemolysis Schistocyte Purpura Hemolytic anemia Coagulation Thrombosis 12 Blood vessel • 13 • 14 Botto m Line: • 15 von Willebrand disea se commonly presents as mucocutaneous bleeding that may occur after dental surgery. It is characterized by prolonged partial thromboplastin time, decrea sed factor VIII level, and reduced ristocetin-induced platelet aggregation. Von Willebrand disease Partial thromboplastin time Factor VIII Platelet Platelet aggregation Thromboplastin

I iii I ;fi 1!1 I•J for year:l 2017 .. FI RST AID FACTS

- · ..... - .. n.,.. .. 6 s 0 lock Suspend End Block Item: 12 of 15 ~ . I • M k <:] t> al ~· ~ QIO: 3857 .l. ar Previous Next lab 'Vfl1ues Notes Calculator

1 intravascular coagulation and thrombotic thrombocytopenic purpura. Disseminated intravascular coagulation Thrombotic thrombocytopenic purpura Hemolysis Schistocyte Purpura Hemolytic anemia Coagulation Thrombosis 2 Blood vessel 3 4 Bottom Line: 5 von Willebrand disease commonly presents as mucocutaneous bleeding that may occur after dental surgery. It is characterized by prolonged partial 6 thromboplastin time, decreased factor VIII level, and reduced ristocetin-induced platelet aggregation. Von Willebrand disease Partial thromboplastin time Factor VIII Platelet Platelet aggregation Thromboplastin 7

8 9 I iii I ;fi 1!1 I•J for year:l 2017 .. 10 FI RST AI D FACTS

11 12 FA17 p 406.1 Mixed platelet and coagulation disorders • 13 DISORDER PC BT PT PTT MECHANISM AND COMMENTS • 14 von Willebrand t - It Intrinsic pathway coagulation defect: l vWF • 15 disease ..... t PTT (vWF acts to carry/p rotect factor V JII). Defect in platelet plug formati on: l vWF ..... defect in platelet-to-vWF' adhesion. Autosomal dominant. lild but most common inherited bleeding disorder. o platelet aggregation with ri stocetin cofactor assay. Treatment: desmopressin, which releases 6 s 0 lock Suspend End Block Item: 12 of 15 ~ . I • M k <:] t> al ~· ~ QIO: 3857 .l. ar Previous Next lab 'Vfl1ues Notes Calculator - - 1 Transfusion (STOP Making New T hrombi). 2 Labs: schistocyt es, t fibrin degradation 3 products (o-dimers), ~ fibrinogen, ~ factors V 4 and VII I. 5

6 FA17 p 404.2 7 Coagulation disorders PT- tests function of common and extrinsic pathway (factors I, II, V, VII, and X). Defect -+ t PT 8 I lR (i nternational normali zed ratio)-calculated from PT. I = normal,> I = prolonged. Most 9 common test used to follow patients on warfarin. 10 PTT- tests functi on of common and intrinsic pathway (all factors except VII and XI II ). Defect - t 11 P1T Coagulation disorders can be due to clotting factor deficiencies or acquired inhibitors. Diagnosed 12 with a mixing study, in which normal plasma is added to patient's plasma. Clotting factor • 13 deficiencies should correct (the PT or P'IT returns to within the appropriate normal range), • 14 whereas factor inhibitors will not correct.

• 15 DISORDER PT PTT MECHANISM AND COMMENTS Hemophilia A, B, o r C t Intrinsic pathway coagulation defect. • A: deficiency of factor VIII ..... t PTT; X-Iinked recessive. • B: deficiency of factor IX ..... t PTT; X-Iinked recessive. • C: defi ciency of factor XI ..... t P'IT; autosomal recessive. Macrohemorrhage in hemophilia-hemarthroses (bleeding into joints, such as knee r.i.'l), easy bruising, bleeding after trauma or surgery (eg, dental procedures). 6 s 0 lock Suspend End Block Item: 12 of 15 ~ . I • M k <:] t> al ~· ~ QIO: 3857 .l. ar Previous Next lab 'Vfl1ues Notes Calculator • 1 FA17 p405.1 2 Platelet disorders Defects in platelet plug formation .... t bleeding time (BT). 3 Platelet abnormalities .... microhemorrhage: mucous membrane bleeding, epistaxis, petechiae, 4 purpura, t bleeding time, possibl y decreased platelet count (PC). 5 DISORDER PC BT MECHA NISMA NDCO MMENTS 6 Bernard-Soulier -/! t Defect in platelet plug forma tion. Large platelets. 7 syndrome l Gplb .... defect in platelet-to-vWF adhesion. 8 Glanzmann t Defect in platelet integrin a 11 b~ 3 {Gpll b/JI Ia) .... defect in platelet-to-platelet 9 thrombasthenia aggregation, and therefore platelet plug formation. Labs: blood smear shows no platelet clumping. 10 Hemolytic-uremic t Characterized by thrombocytopenia, microangiopathic hemolytic anemia, and 11 syndrome acute renal failure. 12 Typical HUS is seen in children, accompanied by diarrhea and commonly • 13 caused by enterohemorrhagic E coli (EHEC) {eg, 0157:H7). HUS in adults • 14 docs not present with diarrhea; EHEC infection not required . • 15 Same spectrum as TIP, with a similar clinical presentation and same initial treatment of plasmapheresis. Immune t Anti-Gpllb/llla antibodies .... splenic macrophage consumption of thrombocytopenia platelet-antibody complex. May be 1° (idiopathic) or zoto autoimmune disorder, viral illness, malignancy, or drug reaction. Labs: t megakaryocytes on bone marrow biopsy. Treatment: steroids, !VIC, splenectomy (for refractory ITP). Thrombotic ! t Inhibition or deficiency of ADAMTS 13 (vWF metalloprotease) • 6 s 0 lock Suspend End Block Item: 13 of 15 ~ 1 • M k -<:J 1>- Jil ~· !:';-~ QIO: 3858 ..L a r Previous Next Lab~lu es Notes Calculat o r

1 • A 64-year-old homeless man presents with a painful, erythematous, laceration on his right calf. He reports having initially injured the area on a shard IA•A] 2 of glass approximately 3 weeks ago, and while the wound stopped bleeding Initially, he reports that it never "scabbed over." on physical examination, there are multiple bruises on his body, petechiae, pallor, and swollen gums. 3

4 What molecular dysfunction is seen in this man's condition? 5 : 6 A. Decreased activity of thyroperoxidase 7 B. Decreased hydroxylation of proline and lysine residues 8 c. Decreased intestinal absorption of ca ldum 9 10 D. Decreased synthesis of methionine

11 E. Decreased transketolase activity 12 • 13 • 14 • 15

a s 8 Lock Suspend End Block Item: 13 of 15 ~ . I • M k <:] t> al ~· ~ QIO: 3858 .l. ar Previous Next lab 'Vfl1ues Notes Calculator

1 The correct a nswer is B. 71% chose this. 2 The patient presents with symptoms of vitamin C deficiency, or scurvy (ie, poor wound hea ling, bruising, petechiae, anemia, and gum changes), likely in 3 the setting of poor nutrition. In collagen synthesis, vitamin C (ascorbic acid) is required as a cofactor in the hydroxylation of proline and lysine into hydroxyproline and hydroxylysine. Without vitamin C, collagen fibers cannot be cross-linked, and the tensile strength is grea tly reduced, resulting in this 4 patient's symptoms. 5 Ascorbic acid Scurvy Collagen Vitamin c Hydroxyproline Cofactor (biochemistry) Petechia Proline lysine Hydroxylysine Anemia Hydroxylation Vitamin Ultimate tensile strength Nutrition Cross-link Wound healing 6 A is not correct. 5 % chose this. 7 Iodine is oxidized and combined with tyrosine by thyroperoxidase, which are important steps in thyroid hormone synthesis. Clinically, iodine deficiency 8 produces hypothyroidism and goiters, but not the connective tissue problems seen in this patient. Hypothyroidism Iodine deficiency Thyroid hormone Thyroid peroxidase Thyroid Iodine Tyrosine Redox Hormone Goitre Connective tissue 9 c is not correct. 9 % chose this. 10 The active form of vitamin D (1,25-dihydroxycholecalciferol) increa ses the intestinal absorption of calcium. Vitamin D deficiency causes rickets (bending of 11 bones) in children, and osteomalacia (softening of bones) in adults, but not the connective tissue problems present in this patient. Osteomalacia Rickets Vitamin 0 Vitamin Connective tissue Calcitriol Hypovitaminosis 0 Calcium 12 D is not correct. 9 % chose this. 13 Vitamin B12 (cobalamin) is a required cofactor for homocysteine methyltransferase to crea te methionine, and for methylmalonyl coenzyme A (CoA) • 14 mutase to form succinyi-CoA. Vitamin B12 deficiency is seen in pernicious anemia and may manifest with macrocytic megaloblastic anemia, neurologic symptoms, and glossitis. • 15 Megaloblastic anemia Pernicious anemia Cobalamin Succinyi-CoA Cofactor (biochemistry) Glossitis B vitamins Methionine Vitamin 812 Homocysteine Macrocytic anemia Anemia Vitamin Coenzyme A Methionine synthase

E is not correct. 6 % chose this.

Vitamin B1 (thiamine pyrophosphate) is a required cofactor for several dehydrogenase enzyme rea ctions: pyruvate dehydrogenase (which links glycolysis to the TCA cycle), a-ketoglutarate dehydrogenase (an enzyme in the TCA cycle), transketolase (an enzyme in the nonoxidative phase of the pentose phosphate pathway), and branched-chain ketoacid dehydrogenase (involved in amino acid metabolism and implicated in maple syrup disea se). Vitamin B1 deficiency may result in beriberi (cardiac and neurologic complications) and/or Wernicke-Korsakoff syndrome (psychiatric and neurologic complications). Wernicke-Korsakoff syndrome Beriberi Pentose phosphate pathway Thiamine pyrophosphate Glycolysis Amino acid Enzyme Thiamine Cofactor (biochemistry) Citric acid cycle Transketolase Pyruvic acid Vitamin 812 B vitamins Pyruvate dehydrogenase Metabolism Pyrophosphate Keto acid Vitamin Phosphate Pentose 6 s 0 lock Suspend End Block Item: 13 of 15 ~ .I • M k <:] t> al ~· ~ QIO: 3858 .l. ar Previous Next lab 'Vfl1ues Notes Calculator

1 E is not correct. 6 % chose this. Vitamin B1 (thiamine pyrophosphate) is a required cofactor for several dehydrogenase enzyme reactions: pyruvate dehydrogenase (which links glycolysis 2 to the TCA cycle), a-ketoglutarate dehydrogenase (an enzyme in the TCA cycle), transketolase (an enzyme in the nonoxidative phase of the pentose phosphate pathway), and branched-chain ketoacid dehydrogenase (involved in amino acid metabolism and implicated in maple syrup disease). Vitamin B 3 1 deficiency may result in beriberi (cardiac and neurologic complications) and/or Wernicke-Korsakoff syndrome (psychiatric and neurologic complications). 4 Wernicke-Korsakoff syndrome Beriberi Pentose phosphate pathway Thiamine pyrophosphate Glycolysis Amino acid Enzyme Thiamine Cofactor (biochemistry) 5 Citric acid cycle Transketolase Pyruvic acid Vitamin 812 B vitamins Pyruvate dehydrogenase Metabolism Pyrophosphate Keto acid Vitamin Phosphate Pentose Enzyme catalysis 6 7 8 Bottom Line: Vitamin c (ascorbic acid) is a required cofactor in collagen synthesis that takes part in the hydroxylation of proline and lysine, an important step that 9 allows the cross-linking of collagen chains, adding to collagen's tensile strength. Vitamin C deficiency results in scurvy (poor wound healing, bruising, 10 petechiae, anemia, and gum changes). Ascorbic acid Scurvy Cofactor (biochemistry) Petechia Proline lysine Collagen Vitamin C Anemia Hydroxylation Vitamin Wound healing Ultimate tensile strength 11 12 13 I iii I ;fi 1!1 I•J for year:l 2017 .. • 14 FI RST AI D FACTS • 15 FA17 p 65.2 Vitamin C (ascorbic acid)

FUNCTION Antioxidant; also facilita tes iron absorption Found in fruits and vegetables. by reducing it to Fe2+ state. ecessary Pronounce "absorbic" acid. for hydroxylation of proline and lysine in Ancillary treatment for methemoglobinemia by collagen synthesis. I ecessary for dopamine red ucing Fe3+ to Fe2+. ~-hydroxyla se, which converts dopamine to 6 s 0 lock Suspend End Block Item: 13 of 15 ~ 1 • M k -<:J 1>- Jil ~· !:';-~ QIO: 3858 ..L a r Previous Next Lab~lu es Notes Calculat o r • 1 EXCESS lausea, vomiting, diarrhea, fatigue, ca lcium 2 oxalate nephrolithiasis. Can t risk of 3 iron toxicity in predisposed individua ls those with transfusions, hereditary 4 (eg, hemoch romalosis). 5 6 FA17 p 46.2 7 Collagen synthesis and structure 8 0 Synthesis-translation of collagen - Jil ~· !:';-~ QIO: 3787 ..L a r Previous Next Lab~lu es Notes Calculat o r

1 • A 40-year-old woman with Crohn disease presents with a tingling sensation In her fingers and toes and a recent history of fatigue. A complete history lA• A] 2 reveals that 3 years ago she underwent resection of her terminal ileum, but since then she has been feeling well and eating a normal diet. Physical examination demonstrates weakness in all four extremities, hyperreflexia, and a positive Romberg sign. CBC count reveals a hematocrit of 22% and a 3 hemoglobin level of 6 g/dl.

4 Mean Serum HyJlersegmented 5 Choice corpuscular folate PMNs on blood volume (fL) level smear? 6 7 A <100 normal no 8 normal no 9 B 100 10 low >100 yes 11 c

12 normal no D >100 13 • 14 >100 normal E yes • 15

Which of the sets of laboratory results in the table is most likely to be seen In this patient?

: A

B

a s 8 Lock Suspend End Block Item: 14 of 15 ~ 1 • M k -<:J 1>- Jil ~· !:';-~ QIO: 3787 ..L a r Previous Next Lab~lu es Notes Calculat o r

1 volume (fL) level smear? •

2 A <100 normal no 3 normal no 4 8 100 5 low 6 c >100 yes 7 normal no 8 D >100

9 >100 normal 10 E yes

11 12 Which of the sets of laboratory results in the table is most likely to be seen In this patient? 13 : • 14 A • 15 B c

0

E

a s 8 Lock Suspend End Block Item: 14 of 15 ~ 1 • M k -<:J 1>- Jil ~· !:';-~ QIO: 3787 ..L a r Prev ious Next Lab~lu es Notes Calculat or • 1 The co rrect answer i s E. 750/o chose this. 2 This patient presents with subacute combined degeneration, a neurologic condition associated with vitamin B12 deficiency that leads to abnormal myelin. Vitamin B12 deficiency causes macrocytic, megaloblastic 3 anemia (mean corpuscular volume > 100 fl) with hypersegmented neutrophlls on blood smear (see image). 4 This patient likely has an isolated vitamin B12 deficiency secondary to surgical resection of the terminal Ileum. When VItamin B12 is ingested, it combines wit h intrinsic factor secreted by the parietal cells in the 5 stomach. This complex is then absorbed in t he terminal ileum. It is likely that she avoided this deficiency for some time due to the large pool of vitamin B12 stored in the liver. Folate deficiency also presents wit h a 6 macrocytic, megaloblastic anemia wit h hypersegment ed neutrophils. However, it Is not associated with neurolog ic problems. In this patient, folate levels would be expected to be normal because loss of the 7 terminal Il eum does not affect the intestinal absorption of folate, and this patient has been eating a 8 normal diet, which should provide adequate folate levels. Megalobla·.tic anemia Mean corpuscular volume Intrinsic factor Myelin Folate deficiency Vitamin Bl2 B vitamins 9 Ileum Blood film Parietal cell Macrocytic anemia Folic acid Anemia

10 Subacute combined degeneration of spinal cord Uver Vitamin Neutrophil Hypersegmented neutrophil 11 Acute {medicine) Femtolitre Neurology Macrocytosis 12 A i s not correct. JO/o chose this. 13 These laboratory values are consistent w ith a microcytic anemia (ie, Iron deficiency, thalassemia, or lea d poisoning). Microcytic anemias are not associated 14 with subacute combined degeneration. They ca n present with fatigue and pallor, and are most often due to some form of blood loss. Microcytic anemia Lead poisoning Thalassemia Pallor Iron deficiency Anemia Fatigue (medical) Subacute combined degeneration of spinal cord Bleeding Iron • 15 B is not correct . 201o chose this. These laboratory values could be from a hea lthy patient o r from someone with a normocytic anemia such as anemia of chronic disea se, autoimmune hemolytic anemia, or anemia following an acute hemorrhage. Anemia of chronic disease Autoimmune hemolytic anemia Hemolytic anemia Anemia Chronic condition Bleeding Autoimmunity Autoimmune disease Normocytic anemia Hemolysis

C is not correct. 150/o chose this. These laboratory values are consistent with folate deficiency. Folate deficiency Is not associated with the neurologic symptoms observed in this patient, and this patient appears to be receiving adequate amounts of folate In her diet. Folate deficiency is most often seen with chronic alcohol use and rn:::aln••l'"l"'l l"lnn a s 8 Lock Suspend End Block Item: 14 of 15 ~ . I • M k <:] t> al ~· ~ QIO: 3787 .l. ar Previous Next Lab'Vfllues Notes Calculator hemolytic anemia, or anemia following an acute hemorrhage. 1 Anemia of chronic disease Autoimmune hemolytic anemia Hemolytic anemia Anemia Chronic condition Bleeding Autoimmunity Autoimmune disease 2 Normocytic anemia Hemolysis

3 c is not correct. 15% chose this. 4 These laboratory va lues are consistent with folate deficiency. Folate deficiency is not associated with the neurologic symptoms observed in this patient, and this patient appea rs to be receiving adequate amounts of folate in her diet . Folate deficiency is most often seen with chronic alcohol use and 5 malnutrition. 6 Folate deficiency Folic acid Malnutrition Alcohol Neurology 7 D is not correct. 5 % chose this. These laboratory va lues are consistent with a macrocytic nonmegaloblastic anemia, which can develop as a result of liver disea se, hypothyroidism, or 8 drugs that impair DNA synthesis. Macrocytic nonmegaloblastic anemias are not associated with subacute combined degeneration. 9 Hypothyroidism Macrocytic anemia Anemia liver Subacute combined degeneration of spinal cord Macrocytosis liver disease DNA replication DNA 10

11 Bottom Line:

12 Vitamin B12, complexed with intrinsic factor; is absorbed in the terminal ileum. Vitamin B12 deficiency manifests as a macrocytic anemia with hypersegmented neutrophils, as well as symptoms of subacute combined degeneration. Folate deficiency, while also producing macrocytic anemia with 13 hypersegmented neutrophils, does not manifest with neurologic symptoms. Intrinsic factor Macrocytic anemia Ileum Folate deficiency Vitamin B12 Folic acid Neutrophil Vitamin B vitamins Anemia 14 Subacute combined degeneration of spinal cord Hypersegmented neutrophil Macrocytosis Neurology • 15

lijl;fiiJI•l toryear:[2017 • ] FI RST AID FACT S

FA17 p65.1

Vitamin 812 (cobalamin) FUNCTION Cofactor for methionine sy nthase (transfers Found in animal products. 6 s 0 lock Suspend End Block Item: 14 of 15 ~ 1 • M k -<:J 1>- Jil ~· !:';-~ QIO: 3787 ..L a r Previous Next Lab~lu es Notes Calculat o r

1 •

2 FA17 p398.1 3 Macrocytic (MCV > 100 fl) anemia 4 DESCRIPTION FINDINGS 5 Megaloblastic anemia impaired DNA synthesis ... matu ration of RBC macrocytosis, hypersegmcntcd 6 nucleus of precursor cells in bone marrow neutroph igls · , lossitis. 7 delayed relati,·c to maturation of cy t op l a~m . 8 9 10

11 a 12 • Folate deficiency Causes: malnutrition (cg, alcoholics), t homocysteine, normal meth)·lmalonic ac id. 13 malabsorption, drugs (eg, methotrexate, o neurologic symptoms (vs B12 defi ciency). 14 trimethopri m, phenytoin), t requirement (cg, • 15 hemolytic anemia, pregnancy).

Vitamin 812 Causes: insufficient intake (eg, veganism), t homocysteine, t methylmalonic acid. (cobalamin) rn a Ia bsorption (cg, Crohn d iscasc), pcrn icious eurologic symptoms: reversible dementia, deficiency anemia, Diphyllobothrium tatum (fi sh subacute combined degenerati on (due to

tapeworm), gastrectomy. involvement of 8 12 in fatty ac id pathways and myel in srnthesis): spi nocerebellar tract, lateral corticospinal tract, dorsal column dys fu nction. Historicallv diagnosed with the Schilling test. a s 8 Lock Suspend End Block Item: 15 of 15 ~ 1 • M k -<:J 1>- Jil ~· !:';-~ QIO: 3860 ..L a r Previous Next Lab~lu es Notes Calculat o r

1 • A 28-year-old African-American man presents with pallor, jaundice, and dark urine 2 days after starting primaquine for his upcoming trip to sub­ 2 Saharan Africa. He recalls an older brother h aving a similar episode prior to his trip to the same region, but not his two sisters. Laboratory tests show: 3 wac count: 8400/mm• 4 Hemoglobin: 12 g/dl 5 Hematocrit: 36% Platelet count: 289,000/mm• 6 7 Production of which of the following molecules is decreased in this condition? 8 9 : A. Free radicals 10

11 B. Galactose-1-phosphate

12 c. Pyruvate 13 D. The reduced form of nicotinamide a denine dinucleotide phosphate 14 E. Tyrosine 0 15

a s 8 Lock Suspend End Block Item: 15 of 15 ~ . I • M k <:] t> al ~· ~ QIO: 3860 .l. ar Previous Next lab 'Vfl1ues Notes Calculator

1

2 The correct a nswer is D. 73% chose this. This patient most likely has a hereditary condition known as glucose 6-phosphate dehydrogenase (G6PD) deficiency. G6PD is an important enzyme in the 3 hexose monophosphate shunt. It reduces nicotinamide adenine dinucleotide phosphate (NADP+ ) to NADPH, which in turn reduces glutathione (GSSG to 4 GSH ) via glutathione reductase. GSH, the reduced form of glutathione, protects against oxidant injury by reducing such compounds as hydrogen peroxide (H20 2 ) to water. RBCs of individuals with G6PD deficiency are susceptible to hemolytic anemia in states of oxidative stress, such as those caused by 5 infections, certain foods (fava bea ns), and drugs (primaquine, chloroquine, sulfonamides, nitrofura ntoins, and others). Primaquine Chloroquine Hydrogen peroxide Glucose-6-phosphate dehydrogenase deficiency Pentose phosphate pathway Oxidative stress Glutathione reductase 6 Glutathione Nicotinamide adenine dinucleotide phosphate Enzyme Glucose 6-phosphate Hemolytic anemia Glucose-6-phosphate dehydrogenase Adenine Anemia 7 Sulfonamide (medicine) Vicia faba Glucose Phosphate Nicotinamide adenine dinucleotide Hexose Nicotinamide Hemolysis Oxidizing agent Hydrogen Peroxide 8 A is not correct. 7% chose this. 9 Production of free radicals would be increa sed in G6PD deficiency due to the lack of reducing agents. Glucose-6-phosphate dehydrogenase deficiency Radical (chemistry) Glucose-6-phosphate dehydrogenase Reducing agent 10 B is not correct. 7% chose this. 11 Ga lactose-1-phosphate is reduced in galactokinase deficiency. 12 Galactokinase deficiency Galactokinase Galactose !-phosphate

13 c is not correct. 9 % chose this. 14 Pyruvate would be reduced with inhibition of pyruva te kinase, an irreversible enzyme of glycolysis. Glycolysis Pyruvic acid Pyruvate kinase Enzyme Kinase 15 E is not correct. 4 % chose this. Tyrosine is reduced in phenylketonuria, a deficiency of phenylalanine hydroxylase. Phenylketonuria Phenylalanine hydroxylase Phenylalanine Tyrosine

Botto m Line: G6PD deficiency results in reduced levels of NADPH . Glucose-6-phosphate dehydrogenase deficiency Glucose-6-phosphate dehydrogenase 6 s 0 lock Suspend End Block Item: 15 of 15 ~ .I • M k <:] t> al ~· ~ QIO: 3860 .l. ar Previous Next lab 'Vfl1ues Notes Calculator y y y y y y 1 •

2 3 Bottom Line: 4 G6PD deficiency results in reduced levels of NADPH . Glucose-6-phosphate dehydrogenase deficiency Glucose-6-phosphate dehydrogenase 5 6 7 Iill ;fi 1!1J•J for year: 2017 " 8 FI RST AID FACTS 9 FA17 p 75.2 10 Glucose-6-phosphate I ADPII is necessary to keep glutath ione X-linked recessive disorder; most common 11 dehydrogenase red uced, which in turn detoxifies free radicals human enzyme deficiency; more prevalent 12 deficiency and peroxides. ~ l ADPH in RBCs leads to among African Americans. t malarial 13 hemolyti c anemia due to poor RBC defense resista nee. 14 against oxidizing agents (eg, fava beans, Heinz bodies-denatured Hemoglobin 15 sulfonamides, primaquine, antituberculosis precipitates with in RBCs due to oxidative drugs). Infection (most common cause) can stress. also precipitate hemolysis; inflammatory Bite cells- result from the phagocytic removal response produces free radicals that diffuse of Heinz bodies by splen ic macrophages. into RBCs, causing ox ida live damage. Think, "Bite into some I leinz ketchup." 2GSH H 0 '"'"'' ~ c '' 6 s 0 lock Suspend End Block 2 6-phosphogluconolactone NADPH GSSG (oxidized) 3 4 FA17p70.1 5 Summary of pathways 6 7 Galactose metabolism 1 8 0 Galactolanase /m1 t 9 '4C osemlil) Galactose B Requires biotin cofactor E) Galactose+phosphate T Requires thiamine cofactor TPr 9 jo uridyltransferase Glycolysis () Irreversible. important po~nt of regulatiOn Galactose-1-phosphate Glucose 10 Is '>'t¥ " 'dC>lr!mJ) Glycogen --....._ E) Hexokulaselglucolanase 11 " .a o 1e 0 HMPshunt 0 Glucose-6-phosphatase UDP-glucose +-- Glucose-1-phosphate .=:::::;- Glucose-6-phosphate -- 6-phosphogluconolactone ) 12 (K~n G1c •Ice dlst:Jstl Glycogenesis glycogenolysiS + 13 0 Glucose-6-phosphate I 1 (:) dehydrogenase fructose-6-phosphate • Ribulose-5-phosphate 14 G) Transketolase Gluconeogenesis Of10 T 15 0 Phosphofructokinase-! Fructose· l.Gir;osphate ~ e Fructos~metabolism (l) fructose-1.6-bisphosphatase (7) fructokinase (c >entiat fructo ur•a) Gtyceraldehyde·3·P __ DHAP - Fructose-1-phosphate - Fructose ® Aldolase B(fructose mtoterance) ·1 ~ e 1 tl) Aldolase 8 ( tVer!, A (m scle) l3·bisphosphoglycerate Glyceraldehyde ~ Tnose phosphate isomerase .! Glycerol lipid metabolism ® Pyruvate kinase 3-phosphogtycerate TI -- a s 8 Lock Suspend End Block